ChaseDream
标题: 【每日逻辑练习第二季】【1-4】 [打印本页]
作者: balapupu 时间: 2011-11-22 21:22
标题: 【每日逻辑练习第二季】【1-4】
新的一期新气象,很高兴看到许多新的战友,逻辑小分队欢迎你们的加入,大家加油哦~~我们一起努力!
【精练】
4. Speaker: Contemporary business firms need to
recognize that avoiding social responsibility
leads to the gradual erosion of power. This is
Davis and Blomstrom’s Iron Law of
Responsibility: “In the long run, those who do
not use power in a manner which society
considers responsible will tend to lose it.” The
law’s application to human institutions
certainly stands confirmed by history. Though
the “long run” may require decades or even
centuries in some instances, society ultimately
acts to reduce power when society thinks it is
not being used responsibly. Therefore, a
business that wishes to retain its power as long
as it can must act responsibly.
Which one of the following statements, if true, most
weakens the speaker’s argument?
(A) Government institutions are as subject to the
Iron Law of Responsibility as business
institutions.
(B) Public relations programs can cause society to
consider an institution socially responsible
even when it is not.
(C) The power of some institutions erodes more
slowly than the power of others, whether they
are socially responsible or not.
(D) Since no institution is eternal, every business
will eventually fail.
(E) Some businesses that have used power in
socially responsible ways have lost it.
---bible-weaken
【逻辑链}】
1.prep 07-13.
Which of the following most logically completes the argument?
United States manufacturers currently produce most of the world's solar-power generators--most of which are exported to Europe. However, European manufacturers are emerging and probably will ultimately capture much of the European market. The United States government is funding initiatives intended to encourage use of solar power within the United States. If these initiatives succeed in increasing the demand for solar-power generators in the United States, United States manufacturers will probably maintain significant production levels, since __________.
(A) some United States manufacturers have been substantially increasing their output over the last several years
(B) the efficiency of solar-power generators in converting energy from the Sun into electric power is not improving as fast as it once did
(C) just as European manufacturers enjoy certain competitive advantages in Europe, so do United States manufacturers in the United States
(D) European governments are currently undertaking initiatives to stimulate the use of solar power within Europe
(E) the current market for solar-power generators in the United States is very limited
2.prep 07-14.
The higher the level of certain vitamins and minerals in the bloodstream, the better a person's lung function, as measured by the amount of air the person can expel in one second. The lung function of smokers is significantly worse, on average, than that of nonsmokers. Clearly, therefore, one way for smokers to improve their lung function is for them to increase their intake of foods that are rich in these helpful vitamins and minerals.
Which of the following is an assumption on which this argument depends?
(A) Smokers are less likely than nonsmokers to have diets that are rich in vitamins and minerals.
(B) The lung function of smokers whose diets are rich in those vitamins and minerals is generally better than that of nonsmokers with comparable diets.
(C) People whose diets are deficient in those vitamins and minerals do not typically have other health problems in addition to diminished lung function.
(D) Stopping smoking will not typically improve lung function more than any diet changes can.
(E) Smoking does not introduce into the body chemicals that prevent the helpful vitamins and minerals from entering the bloodstream.
3.prep 07-15.
Astronomer: Observations of the Shoemaker-Levi comet on its collision course with Jupiter showed that the comet broke into fragments before entering Jupiter's atmosphere in 1994, but they did not show how big those fragments were. In hopes of gaining some indication of the fragments' size, astronomers studied spectrographic analyses of Jupiter's outer atmosphere. These analyses revealed unprecedented traces of sulfur after the fragments' entry. The fragments themselves almost certainly contained no sulfur, but many astronomers believe that the cloud layer below Jupiter's outer atmosphere does contain sulfur. Since sulfur would have seeped into the outer atmosphere if comet fragments had penetrated this cloud layer, it is likely that some of the fragments were at least large enough to have passed through Jupiter's outer atmosphere without being burned up.
In the astronomer's argument, the two portions in boldface play which of the following roles?
(A) The first presents a circumstance for which the astronomer offers an explanation; the second is part of that explanation.
(B) The first acknowledges a consideration that weighs against the conclusion of the argument; the second is that conclusion.
(C) The first acknowledges a consideration that weighs against the conclusion of the argument; the second provides evidence in support of that conclusion.
(D) The first provides evidence in support of the conclusion of the argument; the second acknowledges a consideration that weighs against that conclusion.
(E) The first is a judgment advanced in support of the conclusion of the argument; the second is that conclusion.
4.prep 07-16. (27621-!-item-!-188;#058&003223)
Most of the world's supply of uranium currently comes from mines. It is possible to extract uranium from seawater, but the cost of doing so is greater than the price that uranium fetches on the world market. Therefore, until the cost of extracting uranium from seawater can somehow be reduced, this method of obtaining uranium is unlikely to be commercially viable.
Which of the following would it be most useful to determine in evaluating the argument?
(A) Whether the uranium in deposits on land is rapidly being depleted
(B) Whether most uranium is used near where it is mined
(C) Whether there are any technological advances that show promise of reducing the cost of extracting uranium from seawater
(D) Whether the total amount of uranium in seawater is significantly greater than the total amount of uranium on land
(E) Whether uranium can be extracted from freshwater at a cost similar to the cost of extracting it from seawater
作者: balapupu 时间: 2011-11-22 21:28
answer keys:
4.This problem is similar in form to the carpet market problem. The conclusion
appears at the end and is conditional in nature: “a business that wishes to retain
its power as long as it can must act responsibly.”
This relationship can be diagrammed as:
WRP = business wish(es) to retain power as long as possible
AR = act responsibly
WRP AR
Hopefully, you identified this conclusion as conditional when you read the
stimulus. As you read the question stem, you should have immediately
prephrased an answer that would allow the sufficient condition to occur without
the necessary condition, namely that a business that wishes to retain power does
not necessarily have to act responsibly. Let us examine the answer choices with
this idea in mind:
Answer choice (A): Because this answer addresses government institutions, this
cannot hurt the conclusion, which is about businesses. If anything, this may
slightly support the argument. In the middle of the stimulus, the Speaker
mentions that “The law’s application to human institutions certainly stands
confirmed by history.” This answer affirms that statement by adding
governments to the named list of human institutions.
Answer choice (B): This is the correct answer. If a public relations program can
cause society to think an institution is socially responsible even when it is not,
then an institution that wishes to retain power could act irresponsibly and then
get a public relations firm to cover up the activities. In this way, the institution
could wish to retain power but not act responsibly. Since this scenario allows
the sufficient condition to occur without the necessary, this weakens the
argument.
Answer choice (C): Many students hold this answer choice as a Contender. The
answer is incorrect because the stimulus contemplates varying rates of power
retention, especially between socially responsible and non-socially responsible
institutions. If you read this answer thinking that the stimulus indicated socially
responsible institutions do not lose power if socially responsible, then you made
a quasi-Mistaken Reversal of the stimulus. There is never a presumption in the
argument that power can be held indefinitely. If there were, this answer would
be much more attractive.
Answer choice (D): The conclusion is clear in saying, “a business that wishes to
retain power as long as it can...” The italicized phrase allows for the idea that
businesses will eventually lose power and ultimately fail. Thus, this answer
does not hurt the argument.
185 Chapter Seven: Weaken Questions
Answer choice (E): This is another attractive answer, and one that lured in
many test takers. The answer states that even though some businesses acted
responsibly (AR), they did not retain power (RP). If this difference between
retaining power and wishing to retain power (WRP) is ignored, then this answer
can be seen as attacking the Mistaken Reversal of the conclusion. As you
learned from the discussion of answer choice (C) of the carpet market question,
attacking the Mistaken Reversal of the conclusion does not hurt the conclusion.
However, this answer is attractive because not only does it address elements of
the conclusion, it also appears as the final answer choice. A test taker who did
not like any of the earlier answers would find this answer quite attractive.
逻辑链:13. C 14. E 15. E 16. A
作者: xeyyxzty 时间: 2011-11-22 21:33
1.background information:Contemporary business firms need to recognize that avoiding social responsibility leads to the gradual erosion of power. “In the long run, those who do not use power in a manner which society considers responsible will tend to lose it.” The law’s application to human institutions certainly stands confirmed by history.
premise: Though the “long run” may require decades or even centuries in some instances, society ultimately acts to reduce power when society thinks it is not being used responsibly.
conclusion: a business that wishes to retain its power as long as it can must act responsibly.
the erosion of power & the lost of the power
(A) Government institutions are as subject to the Iron Law of Responsibility as business institutions.
--support
(B) Public relations programs can cause society to consider an institution socially responsible even when it is not.
--public relations programs--irrelevant--right
(C) The power of some institutions erodes more slowly than the power of others, whether they are socially responsible or not.
--some institutions--what institutions?--maybe it has effect on the conclusion, maybe not
(D) Since no institution is eternal, every business will eventually fail.
--irrelevant
(E) Some businesses that have used power in socially responsible ways have lost it.
--weaken--right
又错了~~~思路完全错误
society ultimately acts to reduce power when society thinks it is not being used responsibly
retain power & wish to retain power
2.background information:United States manufacturers currently produce most of the world's solar-power generators--most of which are exported to Europe. However, European manufacturers are emerging and probably will ultimately capture much of the European market. The United States government is funding initiatives intended to encourage use of solar power within the United States.
premise: If these initiatives succeed in increasing the demand for solar-power generators in the United States, ???
conclusion:United States manufacturers will probably maintain significant production levels,
there are more Americans than Europeans using solar-power generators~when less Europeans use the American made generators, more Americans do.
(A) some United States manufacturers have been substantially increasing their output over the last several years
--the output in the past is irrelevant
(B) the efficiency of solar-power generators in converting energy from the Sun into electric power is not improving as fast as it once did
--the efficiency of generators is irrelevant
(C) just as European manufacturers enjoy certain competitive advantages in Europe, so do United States manufacturers in the United States
--the market is big, and competitors are not as competitive as those in the US--right
(D) European governments are currently undertaking initiatives to stimulate the use of solar power within Europe
--the question is about the demand in the US, not about it in Europe--irrelevant
(E) the current market for solar-power generators in the United States is very limited
--the initiatives have already succeeded in increasing the demand--irrelevant
3.background information:The higher the level of certain vitamins and minerals in the bloodstream, the better a person's lung function, as measured by the amount of air the person can expel in one second. The lung function of smokers is significantly worse, on average, than that of nonsmokers.
premise:??? + increase their intake of foods that are rich in these helpful vitamins and minerals.
conclusion:smokers improve their lung function
can the smokers absorb the vitamins and minerals as much as they eat~
(A) Smokers are less likely than nonsmokers to have diets that are rich in vitamins and minerals.
--weaken
(B) The lung function of smokers whose diets are rich in those vitamins and minerals is generally better than that of nonsmokers with comparable diets.
--support
(C) People whose diets are deficient in those vitamins and minerals do not typically have other health problems in addition to diminished lung function.
--other health problems are irrelevant
(D) Stopping smoking will not typically improve lung function more than any diet changes can.
--stop smoking is irrelevant
(E) Smoking does not introduce into the body chemicals that prevent the helpful vitamins and minerals from entering the bloodstream.
--right
4.background information:Observations of the Shoemaker-Levi comet on its collision course with Jupiter showed that the comet broke into fragments before entering Jupiter's atmosphere in 1994, but they did not show how big those fragments were. In hopes of gaining some indication of the fragments' size, astronomers studied spectrographic analyses of Jupiter's outer atmosphere. These analyses revealed unprecedented traces of sulfur after the fragments' entry. The fragments themselves almost certainly contained no sulfur, but many astronomers believe that the cloud layer below Jupiter's outer atmosphere does contain sulfur.
opposing viewpoints:Observations of the Shoemaker-Levi comet on its collision course with Jupiter showed that the comet broke into fragments before entering Jupiter's atmosphere in 1994
premise: sulfur would have seeped into the outer atmosphere if comet fragments had penetrated this cloud layer
conclusion: some of the fragments were at least large enough to have passed through Jupiter's outer atmosphere without being burned up.
(A) The first presents a circumstance for which the astronomer offers an explanation; the second is part of that explanation.
--the second is not part of the astronomer's explanation
(B) The first acknowledges a consideration that weighs against the conclusion of the argument; the second is that conclusion.
--right
(C) The first acknowledges a consideration that weighs against the conclusion of the argument; the second provides evidence in support of that conclusion.
--the second is not evidence.
(D) The first provides evidence in support of the conclusion of the argument; the second acknowledges a consideration that weighs against that conclusion.
--the first is not evidence, and it doesn't support the conclusion.
(E) The first is a judgment advanced in support of the conclusion of the argument; the second is that conclusion.
--the first is truth, not a judgment--right
错了~~~
B--the first doesn't weigh against the conclusion~in fact, the sulfur has nothing to do with the conclusion
5.
Which of the following would it be most useful to determine in evaluating the argument?
premise:Most of the world's supply of uranium currently comes from mines. It is possible to extract uranium from seawater, but the cost of doing so is greater than the price that uranium fetches on the world market.
conclusion:Therefore, until the cost of extracting uranium from seawater can somehow be reduced, this method of obtaining uranium is unlikely to be commercially viable.
maybe some day the costs to extract uranium from mines increases~higher than those from seawater~
(A) Whether the uranium in deposits on land is rapidly being depleted
--if the uranium on land is being depleted, the cost is unnecessary--right
(B) Whether most uranium is used near where it is mined
--the fee for transportation is irrelevant
(C) Whether there are any technological advances that show promise of reducing the cost of extracting uranium from seawater
--irrelevant
(D) Whether the total amount of uranium in seawater is significantly greater than the total amount of uranium on land
--the amount of uranium is irrelevant--the question is about the cost
(E) Whether uranium can be extracted from freshwater at a cost similar to the cost of extracting it from seawater
--the freshwater is irrelevant
作者: chensong211 时间: 2011-11-22 22:32
精炼题:
premise: society ultimately acts to reduce power when it is not being used responsibly.
Conclusion: a business wishes to retain its power as long as it can act responsibly.
A.无关,这里讲institutions 而且没有谈到as long as 的问题。
B.无关,公共关系计划不是business
C.无关,这里没有提到关键词。Business
D. bingo 直接weaken结论。不是act responsibly the business retain its power.
E. 无关
逻辑链
第二题
premise: higher level of vitamins and minerals, the better a person’s lung function. Smokers is worse.
Conclusion: the smokers would better to eat the food that are rich in vitamins and minerals.
Logical: improve the lung function = the rich vitamins and minerals = eat the food that are rich in vitamins and minerals.
A. bingo
其他的分析不是很好。就贴出两题吧。我要看看其他的分析。
作者: fox0923 时间: 2011-11-22 22:46
明天该我了,好激动啊~
占位~~~今天又阴天,情绪小低落~
----------------------------------------分界线------------------------------------------------------
精练-----------1min----------------weaken
B: According to B of D's law, the business will loss power unless it recognizes the social responsibility.
P: The society will reduce the business' power if it thinks the business does not act responsibly.
C: So the business can retain its power as long as it acts responsibly.
Prephrase: the social responsibility is vary based on different opinions and thoughts of people.
Other reasons can also affect the business' power.
Analysis:
(A) Government institutions are as subject to the
Iron Law of Responsibility as business
institutions.
------------------------------------------government institutions is not mentioned in the passage. so this is irrelevant.
(B) Public relations programs can cause society to
consider an institution socially responsible
even when it is not.
------------------------------------------public relations programs affect the society to change the standard of social responsibility. This can be a contender.
(C) The power of some institutions erodes more
slowly than the power of others, whether they
are socially responsible or not.
------------------------------------------the conclusion talks about the act of socially responsible will determine the power of business, but the choice here makes a comparison between socially responsible business and non-socially responsible business. This doesn't weaken the conclusion.
(D) Since no institution is eternal, every business
will eventually fail.
-----------------------------------------this can be a support choice.
(E) Some businesses that have used power in
socially responsible ways have lost it.
------------------------------------------举出一个范例说明即使是遵循了social responsibility,也还是会loss power。
齐擦擦又错了~~不知道是B还是E?倾向于B,但觉得B有点过于局限性--因为说了public relation programs.那其他的呢?还是我想多了?
分析:
B:If a public relations program can
cause society to think an institution is socially responsible even when it is not,
then an institution that wishes to retain power could act irresponsibly and then
get a public relations firm to cover up the activities.----这么说看来是我想多了,这个选项本身用了一种假设的情况,所以依照这种假设情况就已经可以weaken了~~
E: As you learned from the discussion of answer choice (C) of the carpet market question, attacking the Mistaken Reversal of the conclusion does not hurt the conclusion. However, this answer is attractive because not only does it address elements of the conclusion, it also appears as the final answer choice. A test taker who did not like any of the earlier answers would find this answer quite attractive.
----~~原来B-->非A,是不可以weaken A--〉B的,不知道是不是这样理解的?
The answer states that even though some businesses acted responsibly (AR), they did not retain power (RP).谁能给解释一下上面的reversal of the conclusion? 用AR的business还是有可能会loss power的,不明白怎么reversal了?
This relationship can be diagrammed as:
WRP = business wish(es) to retain power as long as possible
AR = act responsibly
WRP AR
作者: zz42050524 时间: 2011-11-22 23:40
巴拉今天发啊,明天来做
作者: 小意达de花儿 时间: 2011-11-23 11:11
占楼,囧,昨天的忘记做了,今天一起补上
作者: Jane412 时间: 2011-11-23 19:33
这就做。。
作者: CHRISTINE2010 时间: 2011-11-23 19:50
每天都补作业~~
作者: Jane412 时间: 2011-11-23 19:51
精练:[weaken]time:42’‘
premise: avoid social responsibility leads to the gradual erosion of power
premise:business must act responsibly
conclusion: business can retain its power
premise: act responsibly 不是唯一retain power的方法。或者二者没必然关系
A) Government institutions are as subject to the
Iron Law of Responsibility as business
institutions.---------------------------------------------------------------题目和business有关,没有涉及government,irrelevant
(B) Public relations programs can cause society to
consider an institution socially responsible
even when it is not.
--------------------------------------------------------------目的是要retain power,不是consider sth responsible,irrelevant
(C) The power of some institutions erodes more
slowly than the power of others, whether they
are socially responsible or not.
--------------------------------------------------------------irrelevant。题目是retain power和responsible的关系。这里是erode power和responsible无关。两个说的不是一个事儿,所以不对。
(D) Since no institution is eternal, every business
will eventually fail.
--------------------------------------------------------------题目是as long as it can.本来就没说eternal,irrelevant。
(E) Some businesses that have used power in
socially responsible ways have lost it.
---------------------------------------correct!说明responsible不能retain power
作者: Jane412 时间: 2011-11-23 19:51
1。[support]time:31''
premise: these initiatives succeed in increasing the demand for solar-power generators in us
conclusion: us will maintain significant production levels
premise: 欧洲本国生产商占领欧洲市场-------》美国送往欧洲的能源需求下降
要想保证美国生产水平不受影响,加大本国的能源使用《------------------美国本土的使用量能够弥补欧洲市场下降的使用量
(A) some United States manufacturers have been substantially increasing their output over the last several years
-------------------------------------------------无关选项。
(B) the efficiency of solar-power generators in converting energy from the Sun into electric power is not improving as fast as it once did
------------------------------------------------能源转化效率和使用量题目中没有涉及,irrelevant
(C) just as European manufacturers enjoy certain competitive advantages in Europe, so do United States manufacturers in the United States
-------------------------------------------------correct。本国市场有竞争力,那么就能保持水平
(D) European governments are currently undertaking initiatives to stimulate the use of solar power within Europe
-------------------------------------------------只能得出美国能源在欧洲市场使用量受损,但和美国本土使用量无关。irrelevant
(E) the current market for solar-power generators in the United States is very limited
--------------------------------------------------weaken。市场受限,那么生产水平会下降。
2.[support]
time:27''
premise: vitamins and minerals are good for lung function
premise: lung function of smokers is worse than that of nonsmokers
premise: increase intake of foods that are full of vitamins and minerals
conclusion: the lung function of smokers will be improved
prephrase: 这招儿适用于所有人包括吸烟这种情况。吸烟不会对营养的吸收造成负作用。
(A) Smokers are less likely than nonsmokers to have diets that are rich in vitamins and minerals.
-----------------------weaken。smokers不吃这些食物,肺功能应该是worse
(B) The lung function of smokers whose diets are rich in those vitamins and minerals is generally better than that of nonsmokers with comparable diets.
-----------------------irrelevant。这个是说吸烟对肺功能有好处了。
(C) People whose diets are deficient in those vitamins and minerals do not typically have other health problems in addition to diminished lung function.
-----------------------irrelevant。这里是说这类食物和其他疾病的关系了。
(D) Stopping smoking will not typically improve lung function more than any diet changes can.
-----------------------irrelevant。这里是说吸烟和肺功能下降的关系了。
(E) Smoking does not introduce into the body chemicals that prevent the helpful vitamins and minerals from entering the bloodstream.
-----------------------------------correct。
3.
time:57''
prephrase: 科学家想知道这些fragment的大小,然后研究发现了sulfur出现
fragment本身没有sulfur,但是out atmosphere下面的云层有硫磺。---------------》premise
云层穿透后硫磺会沉入外层大气。
some fragments were large enough blabla-----------------》conclusion
(A) The first presents a circumstance for which the astronomer offers an explanation; the second is part of that explanation.
-----------解释不通
(B) The first acknowledges a consideration that weighs against the conclusion of the argument; the second is that conclusion.
-----------二者是前提和结论的关系
(C) The first acknowledges a consideration that weighs against the conclusion of the argument; the second provides evidence in support of that conclusion.
-----------不对。第一个不是反驳。第二个是结论
(D) The first provides evidence in support of the conclusion of the argument; the second acknowledges a consideration that weighs against that conclusion.
-------------第二个是结论
(E) The first is a judgment advanced in support of the conclusion of the argument; the second is that conclusion.
--------------correct
这类holdface题是不是自己事先把逻辑关系理顺了,就像我前面prephrase那样?
4。[support]
premise:大部分的uranium从mines中提取
premise: 从seawater中提取uranium的成本>市场上的卖价
premise:海水提取的成本不下降
conclusion:这种海水提取uranium的方法没有经济价值
prephrase:还有别的因素影响是否使用这种方法
A) Whether the uranium in deposits on land is rapidly being depleted
----------------------correct。陆上uranium量不多了,那么不论成本,海水方法都要使用了
(B) Whether most uranium is used near where it is mined
----------------------结论和mine无关。irrelevant
(C) Whether there are any technological advances that show promise of reducing the cost of extracting uranium from seawater
---------------------这个不知道怎么解释排除。。。
(D) Whether the total amount of uranium in seawater is significantly greater than the total amount of uranium on land
----------------------无关
(E) Whether uranium can be extracted from freshwater at a cost similar to the cost of extracting it from seawater
-----------------------------------无关
作者: qiuhua01234567 时间: 2011-11-23 20:59
标题: fox please help me ,question 5
占位先,我今天等了一个白天了- -
1weaken 44S
Premise:Though the “long run” may require decades or even centuries in some instances, society ultimately acts to reduce power when society thinks it is not being used responsibly.
Conclusion:a business that wishes to retain its power as long as it can must act responsibly.
A)Government institutions are as subject to the
Iron Law of Responsibility as business
institutions.
-----------------------------------------------------irrelevent
(B) Public relations programs can cause society to
consider an institution socially responsible
even when it is not.
------------------------------------------------------irrelevent
(C) The power of some institutions erodes more
slowly than the power of others, whether they
are socially responsible or not.
---------------------------------------------------------irrelevent
(D) Since no institution is eternal, every business
will eventually fail.
---------------------------------------------------------irrelevent
(E) Some businesses that have used power in
socially responsible ways have lost it.
---------------------------------------------------------correct
2.25Sassumption
Background: United States manufacturers currently produce most of the world's solar-power generators--most of which are exported to Europe
. However, European manufacturers are emerging and probably will ultimately capture much of the European market.
Ways: funding initiatives intended to encourage use of solar power within the United States
Conclusion: United States
manufacturers will probably maintain significant production levels
(A) some United States
manufacturers have been substantially increasing their output over the last several years
-------------------------------------------------------------------------------------------------irrelevent
(B) the efficiency of solar-power generators in converting energy from the Sun into electric power is not improving as fast as it once did
--------------------------------------------------------------------------------------------------irrelevent
(C) just as European manufacturers enjoy certain competitive advantages in Europe, so do United States manufacturers in the United States
---------------------------------------------------------------------------------------------------irrelevent
(D) European governments are currently undertaking initiatives to stimulate the use of solar power within Europe
-----------------------------------------------------------------------------------------------------irrelevent
(E) the current market for solar-power generators in the United States
is very limited
---------------------------------------------------------------------------------------------------------correct.
3.27Sassumption
Background: The higher the level of certain vitamins and minerals in the bloodstream, the better a person's lung function, as measured by the amount of air the person can expel in one second. The lung function of smokers is significantly worse, on average, than that of nonsmokers
Premise: increase their intake of foods that are rich in these helpful vitamins and minerals
Conclusion: smokers to improve their lung function
(A) Smokers are less likely than nonsmokers to have diets that are rich in vitamins and minerals.
-----------------------------------------------------------------------------------------a little weaken
(B) The lung function of smokers whose diets are rich in those vitamins and minerals is generally better than that of nonsmokers with comparable diets.
--------------------------------------------------------------------------------------------irrelevent
(C) People whose diets are deficient in those vitamins and minerals do not typically have other health problems in addition to diminished lung function.
--------------------------------------------------------------------------------------------irrelevent.
(D) Stopping smoking will anot typically improve lung function more than any diaet changes can.
-----------------------------------------------------------------------------------------irrelevent
(E) Smoking does not introduce into the body chemicals that prevent the helpful vitamins and minerals from entering the bloodstream.
-----------------------------------------------------------------------------------------------correct
445S blodface
(A) The first presents a circumstance for which the astronomer offers an explanation; the second is part of that explanation.
(B) The first acknowledges a consideration that weighs against the conclusion of the argument; the second is that conclusion.
(C) The first acknowledges a consideration that weighs against the conclusion of the argument; the second provides evidence in support of that conclusion.
(D) The first provides evidence in support of the conclusion of the argument; the second acknowledges a consideration that weighs against that conclusion.
(E) The first is a judgment advanced in support of the conclusion of the argument; the second is that conclusion.
--------------------------------------------------------------------------------------------correct
5.evalution 25S
Premise:It is possible to extract uranium from seawater, but the cost of doing so is greater than the price that uranium fetches on the world market.
Conclusion:until the cost of extracting uranium from seawater can somehow be reduced, this method of obtaining uranium is unlikely to be commercially viable.
(A) Whether the uranium in deposits on land is rapidly being depleted
(B) Whether most uranium is used near where it is mined
(C) Whether there are any technological advances that show promise of reducing the cost of extracting uranium from seawater
(D) Whether the total amount of uranium in seawater is significantly greater than the total amount of uranium on land
(E) Whether uranium can be extracted from freshwater at a cost similar to the cost of extracting it from seawater
作者: balapupu 时间: 2011-11-23 21:15
1.[1'12']
P:speaker:Company avoid using responsibility will tend to lose its power.-->B and D law prove that "in the long run, this statement is right.-->Though may require a long time to prove,the society will reduce the power of the company that take responsibility.
C:A company wants to retain its power must take responsibility.
Weaken: there are other measures that company can improve the losing power that result from not taking responsibility.
A:
A) Government institutions are as subject to the
Iron Law of Responsibility as business
institutions.-->support
(B) Public relations programs can cause society to
consider an institution socially responsible
even when it is not.-->the passage mainly about the already lost responsibility.[e..又是攻击点想偏了。。]
(C) The power of some institutions erodes more
slowly than the power of others, whether they
are socially responsible or not.-->irrelevant comparison.
(D) Since no institution is eternal, every business
will eventually fail.-->even the statement is right,have is still better than have not, no weaken.
(E) Some businesses that have used power in
socially responsible ways have lost it.-->R
作者: balapupu 时间: 2011-11-23 21:23
明天该我了,好激动啊~
占位~~~今天又阴天,情绪小低落~
----------------------------------------分界线------------------------------------------------------
精练-----------1min----------------weaken
B: According to B of D's law, the business will loss power unless it recognizes the social responsibility.
P: The society will reduce the business' power if it thinks the business does not act responsibly.
C: So the business can retain its power as long as it acts responsibly.
Prephrase: the social responsibility is vary based on different opinions and thoughts of people.
Other reasons can also affect the business' power.
Analysis:
(A) Government institutions are as subject to the
Iron Law of Responsibility as business
institutions.
------------------------------------------government institutions is not mentioned in the passage. so this is irrelevant.
(B) Public relations programs can cause society to
consider an institution socially responsible
even when it is not.
------------------------------------------public relations programs affect the society to change the standard of social responsibility. This can be a contender.
(C) The power of some institutions erodes more
slowly than the power of others, whether they
are socially responsible or not.
------------------------------------------the conclusion talks about the act of socially responsible will determine the power of business, but the choice here makes a comparison between socially responsible business and non-socially responsible business. This doesn't weaken the conclusion.
(D) Since no institution is eternal, every business
will eventually fail.
-----------------------------------------this can be a support choice.
(E) Some businesses that have used power in
socially responsible ways have lost it.------------------------------------------举出一个范例说明即使是遵循了social responsibility,也还是会loss power。
齐擦擦又错了~~不知道是B还是E?倾向于B,但觉得B有点过于局限性--因为说了public relation programs.那其他的呢?还是我想多了?
分析:
B:If a public relations program cancause society to think an institution is socially responsible even when it is not,then an institution that wishes to retain power could act irresponsibly and thenget a public relations firm to cover up the activities.----这么说看来是我想多了,这个选项本身用了一种假设的情况,所以依照这种假设情况就已经可以weaken了~~
E: As you learned from the discussion of answer choice (C) of the carpet market question, attacking the Mistaken Reversal of the conclusion does not hurt the conclusion. However, this answer is attractive because not only does it address elements of the conclusion, it also appears as the final answer choice. A test taker who did not like any of the earlier answers would find this answer quite attractive.----~~原来B-->非A,是不可以weaken A--〉B的,不知道是不是这样理解的?The answer states that even though some businesses acted responsibly (AR), they did not retain power (RP).谁能给解释一下上面的reversal of the conclusion? 用AR的business还是有可能会loss power的,不明白怎么reversal了?
This relationship can be diagrammed as:WRP = business wish(es) to retain power as long as possibleAR = act responsiblyWRP AR-- by 会员 fox0923 (2011/11/22 22:46:24)
糊糊,我觉得这里指的是3题的迷惑项和这一道题的迷惑项性质是一样的,都叫做:
Mistaken Reversal of the conclusion, and so the attack is made on a
statement that uses the same terms as the conclusion but puts them in
a different relationship(这一部分是我从第3题的解释中摘过来的)
原来B-->非A,是不可以weaken A--〉B的,不知道是不是这样理解的?
对于这两道题的陷阱方式,我觉得是可以这么理解的,但是我觉得GMAT考试木有这么难,是不是这种思路在GMAT中就应该算是weaken?
作者: balapupu 时间: 2011-11-23 22:06
1.[26s]
P:US export solar energy to the European-->European solar companies are now garbing the market-->US company should divert its point to the US market-->it will help US solar energy company maintain high market level.
Reason:the US market is not fully developed and much larger than the European market.
A:
(A) some United States manufacturers have been substantially increasing their output over the last several years-->last year can not guarantee the future.
(B) the efficiency of solar-power generators in converting energy from the Sun into electric power is not improving as fast as it once did-->irrelevant with the efficiency.
(C) just as European manufacturers enjoy certain competitive advantages in Europe, so do United States manufacturers in the United States-->not explain the conclusion.
(D) European governments are currently undertaking initiatives to stimulate the use of solar power within Europe-->the conclusion is about the US.
(E) the current market for solar-power generators in the United States is very limited-->R
[OMG...这道题也错了,谁能给我解释解释,就是因为现在市场小,所以才能说将来的市场前景广阔啊。。。]
2.[30s]
P:higher vitamins and minerals intake-->better lung function.==>nonsmokers-->better lung function.
C:in order to help smokers to maintain better lung function,we should encourage them to take more vitamins and minerals.
assumption:people will not prefer eat Vitamins and minerals because their better lung function.
A:
(A) Smokers are less likely than nonsmokers to have diets that are rich in vitamins and minerals.-->if weaken, not attack the conclusion.
(B) The lung function of smokers whose diets are rich in those vitamins and minerals is generally better than that of nonsmokers with comparable diets.-->repeat the statement in the passage.
(C) People whose diets are deficient in those vitamins and minerals do not typically have other health problems in addition to diminished lung function.-->irrelevant with other health problems.
(D) Stopping smoking will not typically improve lung function more than any diet changes can.-->no mention stopping smoking
(E) Smoking does not introduce into the body chemicals that prevent the helpful vitamins and minerals from entering the bloodstream.-->R
4.[25s]
P:word's supply of U come form mines-->we can extract the U from the sea water-->however, the cost is expensive.-->it is much cheaper than just purchase on the world market.
C:It is unlikely to extract the U from the sea water unless the cost is decreased.
evaluate: whether the world market price of the U will increase.
A:
(A) Whether the uranium in deposits on land is rapidly being depleted-->R
(B) Whether most uranium is used near where it is mined->irrelevant with the location.
(C) Whether there are any technological advances that show promise of reducing the cost of extracting uranium from seawater-->the conclusion is mainly about the cost.
(D) Whether the total amount of uranium in seawater is significantly greater than the total amount of uranium on land--> irrelevant withe the U amount.
(E) Whether uranium can be extracted from freshwater at a cost similar to the cost of extracting it from seawater
-->irrelevant with the location.
作者: winghyy 时间: 2011-11-23 22:10
1. 52S-weaken
Premise:A businees that do not take responsibility will lose its power.Because in xx, it is written that in the long run, those who do
not act responsibly will tend to lose the power.
Conclusion:if a business wants to retain its power, it must act reponsibly.
Prephase:a business can retain its power without acting reponsibly.
E
又错了,让我细细看一下解释吧。。。
作者: corneliaflower 时间: 2011-11-24 00:15
精炼题还在研究中 明天发~
prep07 忘计时了
1 background:US produce solar power and the most exported to the Europe
premise:
1 Europe manafacture now can capture the whole European market
2 US government encourage use solar power within US
conclusion: the US prodution will remain the same
reason:C
2 background: as measured by the amount of the air expelling in one second, a higher lever of vitamin and minerals person has better lung funvtion
premise:smoker's lung function is worse than non-smokers
conclusion: to improve the smoker's function, they should eat more food contain vitamin and minerals
assumption:E
3 方向相同 第二句是结论(分析的好简单...)
E
4 premise: uranium comes from the market are cheaper than from seawater
conclusion: we will not use this method until a way can reduce the cost underway
evaluation:A
大家都分析的好详细 (⊙o⊙)…
作者: fox0923 时间: 2011-11-24 01:18
糊糊,我觉得这里指的是3题的迷惑项和这一道题的迷惑项性质是一样的,都叫做:
Mistaken Reversal of the conclusion, and so the attack is made on a
statement that uses the same terms as the conclusion but puts them in
a different relationship(这一部分是我从第3题的解释中摘过来的)
原来B-->非A,是不可以weaken A--〉B的,不知道是不是这样理解的?
对于这两道题的陷阱方式,我觉得是可以这么理解的,但是我觉得GMAT考试木有这么难,是不是这种思路在GMAT中就应该算是weaken?
bala,就是说把关系倒过来了是不能weaken的,但是不是也有一种情况是A--〉B,然后如果选项由非B--〉非A,好像就是加强还是什么的。记得以前看XDF笔记还是CDers说得,但是这个方法是不是只适用于support而不是weaken?
作者: fox0923 时间: 2011-11-24 01:20
精炼题:
premise: society ultimately acts to reduce power when it is not being used responsibly.
Conclusion: a business wishes to retain its power as long as it can act responsibly.
B.无关,公共关系计划不是business
-- by 会员 chensong211 (2011/11/22 22:32:47)
我跟chensong,熊熊的感觉都一样,上来就irrelevant排出了,没想到是陷阱。可是下次再给我一道题我可能又这么做了。
作者: fox0923 时间: 2011-11-24 02:32
1.[26s]
P:US export solar energy to the European-->European solar companies are now garbing the market-->US company should divert its point to the US market-->it will help US solar energy company maintain high market level.
Reason:the US market is not fully developed and much larger than the European market.
A:
(A) some United States manufacturers have been substantially increasing their output over the last several years-->last year can not guarantee the future.
(B) the efficiency of solar-power generators in converting energy from the Sun into electric power is not improving as fast as it once did-->irrelevant with the efficiency.
(C) just as European manufacturers enjoy certain competitive advantages in Europe, so do United States manufacturers in the United States-->not explain the conclusion.
(D) European governments are currently undertaking initiatives to stimulate the use of solar power within Europe-->the conclusion is about the US.
(E) the current market for solar-power generators in the United States is very limited-->R
[OMG...这道题也错了,谁能给我解释解释,就是因为现在市场小,所以才能说将来的市场前景广阔啊。。。]
我觉得这篇文章不是在说国内市场大小对未来的影响,而是在说两个比较。
1--提到了背景信息,US produces solar power generator出口到欧洲,但是欧洲自己本身的manufactures已经出现并会在未来占领欧洲市场。
2--然后说US开始在本国鼓励solar power generator use.如果这个计划成功,US也会有提高production.
问原因?
那么很显然,US是认为他们本国的manufactures也会像欧洲manufactures一样开始emerging并占领美国市场。
但是E说现在的市场狭小,我反而觉得有weaken的嫌疑,狭小的话,还怎么吸引US manufactures?
作者: fox0923 时间: 2011-11-24 02:36
3.time:57''prephrase: 科学家想知道这些fragment的大小,然后研究发现了sulfur出现 fragment本身没有sulfur,但是out atmosphere下面的云层有硫磺。---------------》premise 云层穿透后硫磺会沉入外层大气。 some fragments were large enough blabla-----------------》conclusion
(A) The first presents a circumstance for which the astronomer offers an explanation; the second is part of that explanation.-----------解释不通
(B) The first acknowledges a consideration that weighs against the conclusion of the argument; the second is that conclusion.-----------二者是前提和结论的关系
(C) The first acknowledges a consideration that weighs against the conclusion of the argument; the second provides evidence in support of that conclusion.-----------不对。第一个不是反驳。第二个是结论
(D) The first provides evidence in support of the conclusion of the argument; the second acknowledges a consideration that weighs against that conclusion.-------------第二个是结论
(E) The first is a judgment advanced in support of the conclusion of the argument; the second is that conclusion.--------------correct
这类holdface题是不是自己事先把逻辑关系理顺了,就像我前面prephrase那样?
-- by 会员 Jane412 (2011/11/23 19:51:56)
对的,我觉得是主要把evidence,conclusion,sub-conclusion,opposed point, proposed point搞明白就好了。内容虽说不重要,但我觉得对于理解逻辑链还是很有帮助的。
作者: prodliu 时间: 2011-11-24 08:40
zuozuokan
作者: zz42050524 时间: 2011-11-24 09:56
2011/11/24
精练:
1. Weaken 59S
P: Contemporay bussiness firms need to realize avoiding responsibility lead to erosion of power. Somebody’s law: In the long run, those who do not use power will lose it. The long run need may be a long time.
C: Bussiness firms wanted as long as they can retain power must act responsible.
推测:1.do not use power wouldn’t lead to lose power.
(A) Government institutions are as subject to the irrelative
Iron Law of Responsibility as business
institutions.
(B) Public relations programs can cause society to support
consider an institution socially responsible
even when it is not.
(C) The power of some institutions erodes more
slowly than the power of others, whether they
are socially responsible or not.
(D) Since no institution is eternal, every business irrelative
will eventually fail.
(E) Some businesses that have used power in
socially responsible ways have lost it.
这道题没有看太明白,最后自己选E。
逻辑链:
2. 45s support
P: Most of the solar engergy generators by US componies are exported to Europe. Eur companies begin to produce those products and will ultimately domin the market. US goverment encourge to consume generators. If US goverment achieve the aim.
C: Manufacturers will maintain the production level.
推测: the potential market of SEG in US is big enough to maintain the production level.
E is just he paraphrase.
3. Method of reasoning 50s
推理连: 提出问题(不知道fragment多大)——提出方法(只有大气层中还有sulfur)——结论(至少fragment足够穿透大气层)
E is the answer.
4. evaluate the argument 35s
P: Most of the U comes from mines. It can also abstract from seawater, but the cost is high.
C: Only the cost is reduced that the method of abstracting from seawater can be use.
推测: 1. The amount of U in mines is whether enough for human beings.
A is the answer
作者: winghyy 时间: 2011-11-24 12:04
逻辑链
1. 54s-complete the argument
BG:Europe is US's major export market of solar-power generators. However, Europen manufactors are emerging and probably will capture the europen market.
Premise:US governement plan to encourage the solar-power use within US, since______
Conlucison: US manufacturers will probably maintain significant production levels
Prephase: more solar consumers in US
C
2. 35s-assumption
BG: the higher level of vitamins and minerals in the bloodstream, the better a person's lung function.
Premise: The lung function of smokers is worse than that of nonsmokers
Conclusion: Smokers should take food containing high level of vitamins and minerals.
Prephase: The good lung function does not lead to high level of vitamins and minerals.
E
3. 65s-bold face [这题读的好纠结]
E
4. 35s-weaken
Premise: U is extracted from mines now. U can also be extracted from seawater, but the cost is very high.
Conclusion: Until the cost of extracting U from seawater reduces, the method is commercially viable.
Prephase: There is no U that can be extracted from mines.
A
作者: balapupu 时间: 2011-11-24 16:33
1.[26s]
P:US export solar energy to the European-->European solar companies are now garbing the market-->US company should divert its point to the US market-->it will help US solar energy company maintain high market level.
Reason:the US market is not fully developed and much larger than the European market.
A:
(A) some United States manufacturers have been substantially increasing their output over the last several years-->last year can not guarantee the future.
(B) the efficiency of solar-power generators in converting energy from the Sun into electric power is not improving as fast as it once did-->irrelevant with the efficiency.
(C) just as European manufacturers enjoy certain competitive advantages in Europe, so do United States manufacturers in the United States-->not explain the conclusion.
(D) European governments are currently undertaking initiatives to stimulate the use of solar power within Europe-->the conclusion is about the US.
(E) the current market for solar-power generators in the United States is very limited-->R
[OMG...这道题也错了,谁能给我解释解释,就是因为现在市场小,所以才能说将来的市场前景广阔啊。。。]
我觉得这篇文章不是在说国内市场大小对未来的影响,而是在说两个比较。
1--提到了背景信息,US produces solar power generator出口到欧洲,但是欧洲自己本身的manufactures已经出现并会在未来占领欧洲市场。
2--然后说US开始在本国鼓励solar power generator use.如果这个计划成功,US也会有提高production.
问原因?
那么很显然,US是认为他们本国的manufactures也会像欧洲manufactures一样开始emerging并占领美国市场。
但是E说现在的市场狭小,我反而觉得有weaken的嫌疑,狭小的话,还怎么吸引US manufactures?
-- by 会员 fox0923 (2011/11/24 2:32:50)
恩恩~~有道理。。。。
作者: 风无衣 时间: 2011-11-24 19:10
【精炼1-4】
weaken_60s
conclusion:a company wants to retain power as long as it can must act responsibly
premise:1、law of responsibility states that in the long run, the company will ultimately loss power when society consider it does not act responsibly.
2、long run may require decades or centuries.
weaken:it is highly possible that the company does not exist before the society find that it does not act responsibly.we should find that a business that wishes to retain power does not necessarily have to act responsibly.
(A) Government institutions are as subject to the
Iron Law of Responsibility as business
institutions.——irrelevant
(B) Public relations programs can cause society to
consider an institution socially responsible
even when it is not.——correct
(C) The power of some institutions erodes more
slowly than the power of others, whether they
are socially responsible or not.——irrelevant
(D) Since no institution is eternal, every business
will eventually fail.——irrelevant
(E) Some businesses that have used power in
socially responsible ways have lost it.
貌似选对了~不过推理思维蛮混乱的~貌似用排除法还比较管用~~><
Answer choice (B): This is the correct answer. If a public relations program can
cause society to think an institution is socially responsible even when it is not,
then an institution that wishes to retain power could act irresponsibly and then
get a public relations firm to cover up the activities. In this way, the institution
could wish to retain power but not act responsibly. Since this scenario allows
the sufficient condition to occur without the necessary, this weakens the
argument.
Answer choice (E): This is another attractive answer, and one that lured in
many test takers. The answer states that even though some businesses acted
responsibly (AR), they did not retain power (RP). If this difference between
retaining power and wishing to retain power (WRP) is ignored, then this answer
can be seen as attacking the Mistaken Reversal of the conclusion.
This relationship can be diagrammed as:
WRP = business wish(es) to retain power as long as possible
AR = act responsibly
WRP AR
【逻辑链1-4】
2、logically complete_45s
米国生产世界上大部分的太阳能generator,其中大部分出口到欧洲~
但是,欧洲的生产商正在整合,最终会占领大部分欧洲市场~
米国政府使用资金激励,鼓励在米国使用太阳能
如果这些激励能够使米国太阳能generator的需求增加,美国生产商可以保持高的生产量。因为……
C
3、assumption_35s
血液里vitamin和mineral的含量越高,人的肺部功能越好
抽烟的人肺部功能比不抽烟的人差
所以,改善抽烟的人的肺部功能的一个办法就是让他们多吃喊vitamin和mineral的食物
E
4、boldface_1min24s
E
5、evaluate_31s
supply of uranium:1、most come from mines 2、extract uranium from seawater——the cost is greater than the price that uranium fetches on the world market
conclusion:the second way of extracting uranium fron seawater is unlikely to be commerically viable unless the cost can be reduced.
A
作者: 风无衣 时间: 2011-11-24 21:40
上来吐槽一下。。。被bible上的题目打击了~~~
作者: corneliaflower 时间: 2011-11-24 22:51
补精练~
花儿一时没反应过来的词汇: Contemporary 当代的 erosion 腐蚀 institutions 习俗,制度
47s
premise:
a law of responsibility tell us in the long run power is close related with responsibility
conclusion:
Business which want to retain its power must take responsibilitiy.
weaken: other reasons may affect power in the long run
B
作者: ugly5552000 时间: 2011-11-25 12:41
1/Background: business firms should realize that avoiding social responsibility leads to erosion of power. Law: those firms who do not use their power in consistent with their responsibility will lose their power.
Premise: a firm wishes to retain its power must act responsibility.
Prephrase: some business firms that act responsibly, but loses its power and bankrupt at last, while some other firms that act irresponsibly still retains power and is prospers.
B
A: government institutions is irrelevant to the business firms
B: even an institution acts irresponsibly, the public relations programs can help to cover the activities, in this way, an institution could retain power even it acts irresponsibly. It weakens the argument.
C: whether the power erodes slowly or not, anyway, if the institution act irresponsibly, its power erodes, support the argument.
D: the argument acknowledges the wish, to retain power as long as they can, of institutions.
E: the argument is a sufficient condition, the answer is a necessary condition.
2/Background: most of world’s solar-power generators are manufactured by Americans. Most of the America-made solar-power generators are exported to Europe. The European manufactures commence to capture European market.
Premise: the US government encourages the civil use of solar power, in order to maintain significant production levels.
Prephrase: the demand within US can not account to that in Europe. The domestic market of energy has little room for solar power.
C
A: the argument is about domestic demand, not output, irrelevant.
B: if solar-power generators have low efficiency, less people will buy them, weaken the argument.
C: when the domestic initiatives increase, the US manufacturers are able to enlarge the market share, as the European manufacturers do in Europe. Support the argument.
D: Europe, irrelevant.
E: weaken the argument.
3/Background: the high level of certain vitamins and minerals in the blood stream makes the lung function better. The lung function of smokers is much worse than that of non-smokers.
Premise: to improve the lung function of lung function, the smokers need to increase their intake of helpful vitamins and minerals.
Prephrase: the digest system of smokers is good enough to support the vitamins and minerals being absorbed by smokers.
E
A: It’s evidence of bad lung function of smokers, not the evidence to the argument.
B: it merely repeats the statement.
C: to smokers, not general people.
D: it has nothing to do with stopping smoking.
E: support
4/Background: because of collision, comet fragments entered Jupiter’s atmosphere. Traces of sulfur were found in the outer atmosphere after the fragment’s entry.
Premise: as the comet fragments did not contain sulfur, many astronomers believeed the cloud layer below outer atmosphere did contain sulfur. There must be some fragments entering the cloud layer, which made the sulfur speed into the outer space.
Prephrase: the 2 portions together can make the argument sufficient.
E
A:if both are explanation, where is the conclusion?
B: not against
C: not against
D: not against
E: the 2 portions together are sufficient to the argument.
5/Background: it is possible to extract uranium from seawater, but the cost is much higher than the current price of uranium on world market.
Premise: only when the cost of extracting uranium from seawater is reduced, can the technology be commercially viable.
Prephrase: the quantity of uranium which could be extracted from seawater is much more than that stored in mines, as the decline of uranium stored in mines, the price of uranium is increasingly rising.
A
A: while the uranium in mines is depleted rapidly, the price will increase, which it is easier for the cost of extracting uranium from seawater to reach.
B: where to use the uranium is irrelevant
C: not sufficient.
D: when the uranium in mine is used up, extracting from seawater will be the only way to continue, then there is no need to reduce the cost.
E: freshwater, irrelevant
作者: trackofheart 时间: 2011-11-26 20:10
4. Speaker: Contemporary business firms need to recognize that avoiding social responsibility leads tothe gradual erosion of power. (背景知识)
This is Davis and Blomstrom’s Iron Law of Responsibility: “In the long run,
those who do not use power in a manner which society considers responsible will tend to lose it.”
The law’s application to human institutions certainly stands confirmed by history.
Though the “long run” may require decades or even centuries in some instances,
society ultimately acts to reduce power when society thinks it is not being used responsibly. (前提)
Therefore, a business that wishes to retain its power as long as it can must act responsibly.(结论)
Which one of the following statements, if true, most weakens the speaker’s argument?
weaken的方法:1、它因导致 2、原因推不出结果 3、直接weaken结论
(A) Government institutions are as subject to the Iron Law of Responsibility as business
institutions.
与结论 a business 要如何无关
(B) Public relations programs can cause society to consider an institution socially
responsible even when it is not.
就是说它不act responsibly,但是Public relations programs同样可以让它显得 responsibly,这样同样可以获得成功。weaken了原文的结论
(C) The power of some institutions erodes more slowly than the power of others,
whether they are socially responsible or not.
erode 的快慢与结论无关
(D) Since no institution is eternal, every business will eventually fail.
institution的永久不永久与结论无关
(E) Some businesses that have used power in socially responsible ways have lost it.
有a little的weaken,但是some不会影响大局,没有b强烈————》与解释的理解不同,要注意选项与原文的小差别,wrp与rp是有区别的,不能将其混同
作者: UlysessHope 时间: 2011-12-30 20:54
【逻辑链}】
1.sentence completion
(A)last several years---irrelevant
(B)irrelevant
(C)link the backgroud to the conclusion
(D)irrelevant
(E)do not rule out the situation that many European manufacturers will capture much of American market in the future
2.assumption
premise:The lung function of smokers is significantly worse than that of nonsmokers. The higher the level of certain vitamins and minerals in the bloodstream, the better a person's lung function.
conclusion: one way for smokers to improve their lung function is for them to increase their intake of foods that are rich in these helpful vitamins and minerals.
strengthen: the vitamins and minerals intaked can be sent to the bloodstream smoothly.
(A) irrelevant
(B) contrary to the background
(C) irrelevant
(D) irrelevant
(E) Smoking does not introduce into the body chemicals that prevent the helpful vitamins and minerals from entering the bloodstream.---hit the point 否定加强
3.boldface
(E) the 2nd BF is the evident conclusion
4.evaluation
(A) uranium is being depleted---- demand surpass supply---the price rises
(B) irrelevant
(C) Whether there are any technological advances that show promise of reducing the cost of extracting uranium from seawater----whether these advances will be used in the extracting industry? we don't know
(D) Whether the total amount of uranium in seawater is significantly greater than the total amount of uranium on land---do not point out the relationship between demand and supply
(E) irrelevant
作者: lan0604 时间: 2012-2-15 00:35
精炼
weaken
power lost in long run without responsibility--->company should behave responsibly
...
B
1
00:32
blank
C
2
00:52
assumption
vitamins and minerals in the bloodstream can make better lung function--->smokers should intake more food with vitamins and minerals
E
3
01:17
bold face
A错误了……E正确
4
00:42
evaluate
only when the cost reduce, method of obtaining is viable
C错误~答案A~貌似我不太懂evaluate的题肿么想吖
作者: Roxas605 时间: 2012-4-1 19:30
【逻辑链】1. Con: Business(wish to retain power)--->act responsibly
Gap: 1.Those who use power in manner =?business that wishes power
2. R=?social R
排除B, weaken GAP2!
boldface做错了。
【总结】:
1. believe that= judgement!
2. since XX,XX= 完整的因果型结论=conclusion!
【继续加油~! pupu~】
作者: leewonting 时间: 2012-4-2 21:32
29 Speaker: Contemporary business firms need to recognize that avoiding social responsibility leads to the gradual erosion of power. This is Davis and Blomstrom’s Iron Law of Responsibility: “In the long run, those who do not use power in a manner which society considers responsible will tend to lose it.” The law’s application to human institutions certainly stands confirmed by history. Though the “long run” may require decades or even centuries in some instances, society ultimately acts to reduce power when society thinks it is not being used responsibly. Therefore, a business that wishes to retain its power as long as it can must act responsibly.
Which one of the following statements, if true, most weakens the speaker’s argument?
(A) Government institutions are as subject to the Iron Law of Responsibility as business institutions.
(B) Public relations programs can cause society to consider an institution socially responsible even when it is not.(公共关系计划能够让社会觉得企业是具备社会责任感的,即使企业其实没有社会责任感)
(C) The power of some institutions erodes more slowly than the power of others, whether they are socially responsible or not.
(D) Since no institution is eternal, every business will eventually fail.
(E) Some businesses that have used power in socially responsible ways have lost it.
背景:In the long run, those who do not use power in a manner which society considers responsible will tend to lose it
条件:Though the “long run” may require decades or even centuries in some instances, society thinks it is not being used responsibly
结论:Therefore, a firm should act responsibly if it wants to retain its power
推测:社会认为是有责任的,是否跟公司的价值观一样?或者有什么反例?
选E,举出反例,即使具备了社会责任,仍然是失败的
应该选B,提出了一种即使社会不进行社会工作,但是通过Public Relation也能让人觉得具备社会责任,就削弱了题目中的一定要act responsibly,E应该是迷惑项,提出act responsibly也会失败,wishing to retain的意思与retaining the power是否完全相同?想要保持权利与保持权利?
30 Which of the following most logically completes the argument?
United States manufacturers currently produce most of the world's solar-power generators--most of which are exported to Europe. However, European manufacturers are emerging and probably will ultimately capture much of the European market. The United States government is funding initiatives intended to encourage use of solar power within the United States. If these initiatives succeed in increasing the demand for solar-power generators in the United States, United States manufacturers will probably maintain significant production levels, since __________.
(A) some United States manufacturers have been substantially increasing their output over the last several years
(B) the efficiency of solar-power generators in converting energy from the Sun into electric power is not improving as fast as it once did
(C) just as European manufacturers enjoy certain competitive advantages in Europe, so do United States manufacturers in the United States
(D) European governments are currently undertaking initiatives to stimulate the use of solar power within Europe
(E) the current market for solar-power generators in the United States is very limited
背景:US manufacturers produce most of the world’s solar-power generators-most of which are exported to Europe
条件:US government is funding initiatives intended to encourage use of solar power within the US
结论:If these initiatives succeed in increasing the demand for solar-power generators in the US, US will maintain significant production levels
推测:the demand of US is same to the demand of Europe?
选E,现在的美国的太阳能市场比较小,如果这些激励能够成功,扩大了市场,那么manufacturers就能继续生产那么多太阳能了
选C,初始想法是“美国现在的市场小,如果美国的激励能使国人更多的使用solar power,市场大了,即使推出欧洲市场,也能保持现在的生产量”,但是题目的切入点应该是US和Europe的相似,美国海外↓(欧洲的生产商在欧洲本地有优势)+美国本土(美国生产商在美国也有优势,通过initiatives找到的)↑=美国生产持平。而且E未必是如我所想的,所以C更加好。
31
背景:The higher the level of certain vitamins and minerals in the bloodstream, the better a person’s lung function, as measured by the amount of air the person can expel in one second
条件:The lung function of smokers is worse than that of the nonsmokers.
结论:Smokers should increase their intake of vitamins and minerals
推测:Lung function is measured only by the amount of air the person can expel and only intake of vitamins and minerals can develop the lung function
选E,吸烟的人不会因为吸烟或吸收进某些阻碍身体吸收维他命和矿物质的化学物质
32
背景:Analyses revealed unprecedented traces of sulfur after the fragments’ entry
条件:The fragments did not contain sulfur but many astronomers believed that the cloud layer below Jupiter’s outer atmosphere dose contain sulfur
结论:some of the fragments were at least large enough to have passed through Jupiter’s outer atmosphere without burned up
推测:
选A
答案好像选E
33
背景:Most of the world’s supply of uranium currently comes from mines
条件:The cost of extracting uranium from seawater is greater than the price of uranium
结论:The method is unlikely to be commercially viable
推测:技术是否提升,或者提取的过程中还有副产品?
选C,与extract uranium from seawater有关
应该选A,如果陆地上面的uranium没有了,怎么都要开采seawater中的uranium了;C的话,就算技术提升了,是否技术的使用令extraction简单了,但是价钱也没有降低,仍然要开采land的。
————————————————————————————————————————————
天啊,只对了一题!
作者: Rena张 时间: 2012-4-6 17:51
啊……全错……不好意思贴出来了都……
作者: zly2011 时间: 2012-4-17 14:24
1、52s weaken
premise society ultimately acts to reduce power when society thinks it is not being used responsibly. ?
conclusion Therefore, a business that wishes to retain its power as long as it can must act responsibly.
推测 even the business act responsibly , other factors will affect its power
选项分析 A government 无关 B 正确答案 C无关比较 D 无关 E 无关
2、58s explanation
premise(推测) USis a big market
conclusion United States manufacturers will probably maintain significant production levels
选项分析 A、无关 B、无关比较 C、无关 D、以 欧洲解释美国 正确答案 E、做错 刚好解释反了
3、29s 假设题
premise The higher the level of certain vitamins and minerals in the bloodstream, the better a person's lung function
conclusion one way for smokers to improve their lung function is for them to increase their intake of foods that are rich in these helpful vitamins and minerals.
推测 the vitamin in the smokers bloodstream is loww
选项分析 A、无关比较 B、无关C、其他疾病 无关 D、无关比较 E、正确
4、30s 答案E
5、29s valuation
premise the cost is great
conclusion until the cost of extracting uranium from seawater can somehow be reduced, this method of obtaining uranium is unlikely to be commercially viable.
推测 uranium 已经没有了
选项分析 A正确答案 B 无关 C无关 D无关比较 E无关比较
作者: corneliaflower 时间: 2012-4-30 16:59
补精练~
花儿一时没反应过来的词汇: Contemporary 当代的 erosion 腐蚀 institutions 习俗,制度
47s
premise:
a law of responsibility tell us in the long run power is close related with responsibility
conclusion:
Business which want to retain its power must take responsibilitiy.
weaken: other reasons may affect power in the long run
B
-- by 会员 corneliaflower (2011/11/24 22:51:13)
今天又做了一遍,错选E。
不明白,求解释。
作者: FB小贝 时间: 2012-5-1 20:32
4.
【精练】Premise: In the long run, those who do
not use power in a manner which society
considers responsible will tend to lose it
Conclusion: a business that wishes to retain its power as long
as it can must act responsibly.
Weaken: does not necessarily have to act resposibly
a. irrelevant
b. irrelevant correct
c. irrelevant
d. irrelevant
e. correct irrelevant
【逻辑链】1.Background: European manufacturers are emerging and probably will ultimately capture much of the European market.
Premise: the demand for spg increases in the US
Conclusion: US manufactures will probably maintain significant production levels
Strengthen: the increase is at least as much as the decrease in the European market./ the emerging manufactures in Europe will not export to the US.
a. irrelevant
b. irrelevant
c. correct
d. irrelevant
e. irrelevant
2.
【前提和结论到底怎么分】Premise: increase their intake of foods that are rich in these helpful vitamins and minerals.
Conclusion: improve the lung functions
Assumption: smoking will not destroy the vitamins and minerals
a. weaken
b. this is conclusion
c. irrelevant
d. irrelevant
e. correct
3. premise.background—conclusion
a. not a circumstance
b. not against
c. not against
d. not against
e. correct
4.Premise: the cost of xxs from seawater s greater than the price that uranium fetches on the world market
Conclusion: until the cost of xxx from seawater can be reduced, this method is unlikely to be commercially viable
Evaluate: the price of u will rise or not
a. correct
b. irrelevant
c. irrelevant
d. irrelevant
e. irrelevant
作者: dwindwin1106 时间: 2012-5-2 05:30
(1)P:comtemperary business companies need to recognize their social responsibility otherwise they will end with the gradual erosion of power.
C:a business company that want to retain its power as long as possible must act responsibly.
Weaken: other factors may lead to the failure of these companies.
修正思路:预测需要针对Conclusion,原文说要长久需act responsibly,weaken就需要说明要长久,不一定需要act responsibly.(即条件型结论,长久->负责,预测答案为当必要条件,即负责,不成立时,充分条件成立,即长久,仍然成立)
Answer: B
A.irrelevant ------ the topic of the argument is business companies not government institutions.
B.Correct ------- the institution that is not socially responsible is able to retain it power since some public relations programs make people think it is a responsible one.
C.Irrelevant ------the comparison is pointless.
D.Irrelevant ------ has no sense on the argument
E.Irrelevant (容易误选) ------ attack the reversal of the conclusion
(2)P:US manufactures currently most of the world’s SYG, most of which are exported to Eu.
 :Eu will ultimately capture much of the Eu market.
 :US is encouraging the use of SYG in US.
C:if the plan is successful, US manufactures will probably maintain their production level
Assumption: the market in US is as large as that in Eu
Answer:C
A.Irrelevant
B.Irrelevant
C.Correct
D.irrelevant
E.Weaken
(3)P:higher level of certain vitamin & minerals in the bloodstream, better a person’s lung function
 :the lung function: smokers<nonsmokers
Cne way to improve the smoker’s lung function is to increase the intake of foods that are rich in these helpful vitamin & minerals.
Assumption: these kind of nutrition in the food can be absorbed into the bloodstream
Answer: E
A.Irrelevant
B.irrelevant
C.Irrelevant
D.irrelevant
E.correct
(4)P:the study revealed unprecedented traces of sulfur after the fragment’s entry to Jupiter’s atmosphere
 :the fragments themselves almost certainly contained no sulfur, but many astro. believe that the cloud layer below Jupiter’s outer atmosphere contain sulfur.
C: Since sulfur can seep into the outer atmosphere if the fragments enter the cloud layer, it is likely that some of the fragments were at least large enough to have passed through Jupiter's outer atmosphere without being burned up.
Boldface
Answer: E
A.First: wrong; second: wrong
B.First: wrong; second: right
C.Both are wrong
D.First: right; second: wrong
E.correct
(5)P:most of U comes from mine. There is a way to extract U from seawater, but cost of extraction is higher than the price U is sold.
C: the extraction of U from seawater is unlikely to be commercially viable until the cost of extraction can reduced.
Evaluation: is there any other reason to make the plan viable?
Answer: A
A.Irrelevant correct
B.Irrelevant
C.Correct irrelevant (等于重复了必要条件,其实表达的还是原文的价格问题?)
D.Irrelevant
E.irrelevant
作者: sherryli 时间: 2012-5-2 18:49
Question 1:
Background: D&B Law
Premise: Though the “long run” may require decades or even centuries in some instances, society ultimately
acts to reduce power when society thinks it is not being used responsibly.
Conclusion: a business that wishes to retain its power as long as it can must act responsibly
If we can prove that even the business act responsibly, it cannot retain its power as long as it can ?
A. Unrelated
b. Unrelated
C. Unrelated; It is not talking about the speed
D. Unrelated; it did not connect the responsibly and power
E. Correct
Question 2:
Premise: US are losing Europe market because of local producer; US govern encourage use of solar power within the US
Because …, so US will probably maintain significant production levels. (because the demand from US local person increased while the lost of Europe market)
A. Unrelated
B. Unrelated
C. Correct. Take the advantage of location
D. Unrelated. We are talking about US
E. Weaken; it cannot support the conclusion
Question 3:
Premise: higher Certain vitamins and minerals, better lung function
Lung functions of smokers worse than that of nonsmokers
Conclusion: for smokers to improve lung is to eat food contain vitams and minerals
Ask for assumption (support)
A. Unrelated
B. Unrelated
C. Unrelated; we did not talk about other health problems
D. Unrelated
E. Correct: If it is not correct, the conclusion is not correct.
Question 4: A
Question 5:
Premise: uranium comes from mines; too expensive to extract uranium from seawater
Conclusion: the method is unlikely to be commercially viable until the cost reduced
Evaluation:
A. Correct. If answers is Yes, the conclusion is not wrong
B. Unrelated
C. Unrelated
D. Unrelated
E. Unrelated
作者: iaoinging 时间: 2012-5-7 10:42
第一篇:1, 44s
2, D and B stated that the people who do not use power in a responsible way will tend to lost it in the long run. And this also confirmed by the law application.
Conclusion: if a business wants to retain its power, then it must act responsibly.
3, we need concern the meaning of “ tend to lost”, if does not mean it must be lost. Moreover, maybe there are some other factors also will influence the ability of a business retains its power. AND what if the social cannot correct to identify the responsibility of a business?
4, B
A, 无关选项。
B,PR也许可能造成一种错觉,让社会认为其公司是有RESPONSIBILITY的,然而其公司实际是没有的。Correct. Because this option successful to stated the flaw of this premise.
C, 无关
D,支持文章的吧
E,阐述一个事实。却没能指出文章的不合逻辑不合理性。
第二篇:1, 38s
2, background: US. companies Manufacture solar-power-generators to exported to Europe. While European manufacturers start to gain markets in solar-power-generators
 remise: US. Government starts to initiatives in order to increase the demand of SPG in US. Hence, US. Companies need not reduce its production level
3. because those companies can sell it inside U.S.
4. C
A, 无关,与其可以保持原有生产量无关系
B,这个讲SPG的不足,应该是反对原文观点的
C,提出了美国公司在美国发展的优势,以证明了计划的可行性。
D,无关
E,反对原文的观点的。
第三篇:1, 30s
2, background: smoker’s lung function is worse than nonsmokers’.
 remise: The higher the level of certain vitamins and minerals in the bloodstream, the better a person's lung function
Conclusion: one way for smokers to improve their lung function is for them to increase their intake of foods that are rich in these helpful vitamins and minerals
3, assumption is a bridge between premise and conclusion. While in this paragraph, the assumption is the smokers did not have a higher level of certain vitamins and minerals so that those people can not have a good lung.
4, E
A, B,C, 因为文章里说的的提高吸烟者LUNG FUNCTION的方法,所以在这里应该跟不吸烟者没有关系,那么A,B,C,都是无关的
D,文章里说的是DIET对吸烟者有用,而这个选项是反对的。
E, 这是唯一一个可以证明文章里的方法可能有效的选项。
第四篇:1, 1.04mins
2, 文章大概是说在1994之前,COMET broke into fragments, then entering J 宇宙;但是现在无法确定那些FRAGMENTS的大小。所以天文学家们又来学习研究某某,发现J宇宙边缘有S物质,但是F没有一点F。 他们得出结论, Since sulfur would have seeped into the outer atmosphere if comet fragments had penetrated this cloud layer, 所以可以大概确定F的大小。
3, 第一句应该是为了得出结论的一个论点。第二个应该就是结论。
4, E
第五篇:1, 30s
2, B: Most of the world's supply of uranium currently comes from mines
 : It is possible to extract uranium from seawater, but the cost of doing so is greater than the price that uranium fetches on the world market.
C: until the cost of extracting uranium from seawater can somehow be reduced, this method of obtaining uranium is unlikely to be commercially viable.
3.有没有其他的原因那些生产者不愿意EXTRACT URANIUM FROM SEAWATER呢。是不是只是因为COST的原因呢。
4,D
A, 赤裸裸的无关选项
B, 这个虽然也牵涉到了COST的问题,可是题目应该是问除了Cost外有没有其他的因素吧。
C, 这个也无关吧,人家题目都说如果Cost可以降下来,是不是可行。
D, 如果他们不用这种方法是因为在海水里能够开采出来的U不多呢。这个必须是个考量点
E, 关系在哪里,亲?
思路错了!!!天哪!!!千万注意文章结论的写法,这里应该是只有COST降下来,这种方法才能用。而不是COST降下来后还有没有其他的考虑。
所以除了COST降下来这种方法,我们要选其他也许能让生产者选择这个方法的情况。那就是A, 因为另一种方法的Cost升高了。
作者: spencerX 时间: 2012-5-13 10:51
B: a company will erode its power if it does not act responsibly. The history of human institution can prove that.
P: the society will reduce the power of companies, if it thinks they are not responsible.
C: A company want to retain its power must be responsible.
Pre: A company actually is very responsible, but it still lose its power./ There is another reason why the company lose its power./ The analogy between companies and human institutions is not proper.
A. This choice actually supports the argument because it prove that the analogy between human institutions and companies is reasonable.
B. Contender. If a company is not responsible but still gain power, then the conclusion may be weakened.
C. “Some companies” cannot stand for “all” companies, so this is an irrelevant comparison.
E. I think this choice is right. Because if a company is responsible but still lose its power, then the conclusion may be weakened.
1.没有注意一个是“wish to retain its power”,另外一个是”retain power”
2.没有发现这里是一个mistaken reversal of the conclusion.
conclusion:wish to retain power→ responsibility
那么reverse一下就是:responsibility→ wish to retain power(假设这里忽略wish的问题)
也就是说“负责”就足以带来“retain power”的结果,选项E中说,负责不足以带来保持实力,等于是weaken了reverse之后的argument,然而weaken mistaken reversal of the conclusion是不能起到weaken原来的argument的作用的。所以错
The Rules of Reversibility
Certain Formal Logic relationships have a natural “reversibility.” Reversibility in the context of Formal Logic means that the relationship between the two variables has exactly the same meaning regardless of which “side” of the relationship is the starting point of your analysis. Statements that are non-reversible have a single “direction,” that is, the relationship between the two variables is not the same.
First, let us examine a relationship that is not reversible:
Starting from the A side, we know that every single A is a B. If we start at B, does the relationship reverse? That is, is every single B an A? No—that would be a Mistaken Reversal. From B’s side, we do not know if every B is an A. Instead, we only know that some B’s are A’s (this inherent inference will be discussed in greater detail in the Inherent versus Additive Inferences section). Thus, the arrow between A and B in the diagram above has a direction: the “all” travels only from A to B and it does not additionally travel from B to A. The relationship is therefore not reversible.
Now, let us examine a reversible relationship. “Some” is a classic example of a reversible statement.
Consider the following example:
Starting from A yields, “Some A’s are B’s” (A some B). Starting from B yields “Some B’s are A’s” (B some A). Because of the nature of “some,” these two statements are functionally identical (if some A’s are B’s, by definition some B’s must also be A’s; alternatively, if some A’s are B’s, then somewhere in the world there is an AB pair, and thus somewhere a B is with an A and we can conclude some B’s are A’s).
Reversible statements are easily identifiable because the relationship symbol is symmetrical and does not include an arrow pointing in a single direction. Non-reversible terms have arrow that point in just one direction.
Reversible Relationships Non-reversible Relationships
The beauty of reversible terms is that you can analyze the relationship from either “side” and still arrive at the same conclusion.
1.B:USA solar- power manufacturers may lose their market share in the European market because European manufacturers are more competitive in Europe
P: USA manufactures succeed in initiate the demand of American
C: They can maintain significant production level
Pre: There is a true analogy between European manufactures and USA ones
2.B: The higher the level of certain vitamins and minerals in the bloodstream, the better a person's lung function
P: Smokers should increase their intake of foods that are rich in vitamins and minerals.
C: The lung function of them will improve.
Pre: The vitamins and minerals taken in by smokers cannot increase the level of vitamins and minerals in the “bloodstream”!
3.B: One observation shows that S comet broken into fragments before entering J. Astronomers studied these fragments.
P: Sulfur would have seeped into the outer atmosphere if comet fragments had penetrated this cloud layer
C: Some of the fragments have not been burned up when they passed through J’s atmosphere.
Pre: statement to support the conclusion / Conclusion
4.B: Most of the world's supply of uranium currently comes from mines.
P: Price of extracting from seawater> fetching on the world market
C: obtaining uranium from seawater is unlikely to be commercially viable →cost of extracting uranium from seawater reduce
Pre: 如果没有reduce,seawater的这种方法是是否仍然viable?
错误原因分析:选项C讨论的是一个现实问题,和能否weaken作者的argument是没有关系的,选项C并不是作者的premise。而选项A中讨论的问题就和前提有关,因为如果陆地上的uranium没有了,人们就不得不转向seawater,在这种情况下,不论挖掘seawater的cost是否会下降,都会使它有商业价值
作者: 卖红薯a 时间: 2012-5-16 21:40
4.
1)计时:80s
2)逻辑链Situation
Background information: Business firms should know that avoiding social responsibility will weak its power.
Premise: e.g. Though the time will be long, society eventually acts to reduce power when society thinks it’s not being used responsibly.
Conclusion: A business must act responsibly if it wants to retain its power as long as it can.
3)推测: A business isn’t the same as a society, and it doesn’t have to act responsibly. Business is business. It has many factors to succeed apart from responsibility, such as product’s quality, quantity, innovation, etc.
4)选项分析:选E
A.Opposite.
B.Irrelevant.
C.Irrelevant.
D.Irrelevant.
E.Right.
1.
1)计时:55s
2)逻辑链Situation
Background information: US produce most of the solar-power generators in the world. However, European producers are ultimately capturing much of European markets.
Premise: The US government is funding initiatives to encourage more use of solar power within US.
Conclusion: If these moves succeed in increasing the demands within US, the producers will maintain the production levels.
3)推测: It gives a great example of success for European in using the US generators. It shows that initiative is a better way to boarder its markets, and it also can guarantee the inner markets in US itself.
4)选项分析:选E
A.Irrelevant.
B.Irrelevant.
C.Irrelevant.
D.Irrelevant.
E.Right.
2.
1)计时:38s
2)逻辑链Situation
Background information: The higher the level of certain vitamins and minerals in the bloodstream, the better a person’s lung function.
Premise: The lung function of a smoker of much worse than that of a nonsmoker.
Conclusion: Increasing their intake of foods which are rich in vitamins and minerals can improve smoker’s lung function.
3)推测: The vitamins and minerals can fully absorbed by smoker’s bodies.
4)选项分析:选E
A.Irrelevant.
B.Irrelevant.
C.Irrelevant.
D.Irrelevant.
E.Right.
3.
1)计时:72s
2)逻辑链Situation
Background information: Observations showed that the SL comet broke into fragments before entering Jupiter’s atmosphere.
Premise: Studies of analyses of Jupiter’s out atmosphere revealed traces of sulfur after fragments’ entry. (boldface) The fragments had no sulfur, but someone believes that the cloud layer did.
Conclusion: Because sulfur would have seeped into the outer atmosphere when fragments entered. (boldface) Some of the fragments were big enough to have passed through the outer atmosphere without burned up.
3)选项分析:选B
A.1. explanation. 2. Part of the explanation.
B.Right. 1. against the conclusion. 2. Conclusion.
C.1. against the conclusion. 2. Evidence of the conclusion.
D.1. support the conclusion. 2. Against the conclusion.
E.1. support the conclusion. 2. conclusion.
4.
1)计时:38s
2)逻辑链Situation
Background information: Most of the supply of uranium comes from mines.
Premise: Extracting uranium from seawater is possible but much more expensive than gaining it from the markets.
Conclusion: This method would be viable if the cost of it can be reduced.
3)推测: It doesn’t mention how much the cost could be reduced.
4)选项分析:选D
A.Irrelevant. even if the uranium is used up on lands, it has nothing to do with the cost of extracting from seawater
B.Irrelevant.
C.Support.
D.Right.
E.Irrelevant. doesn’t mention freshwater
作者: Shirley55555 时间: 2012-5-17 12:14
1. 1'08'' P: ppl lose power if they did not take it. C: so if business take it they would retain it.
W: take it in the wrong would lead to another direction? Choose E ABCD irr.
正确答案:B T.T
2、42'' P: most solar energy USA produce were exported to Euro. Euro begin to produce solar energy bu themselves. USA is enlarging its domestic market. C: If they could enlarge domestic market successfully keep their production level.
Since: the volume enlarged in USA= the volume decline in Euro. Choose A BCDE irr
正确答案 C
3. 50'' P: the higher of V in bloodstream, the better lung function. Smoke's lung function is bad C: smokers can improve their lung function by taking foods rich in V.
assumption: V in foods can transfer to bloodstream. Choose E ABCD irr.
4. 2'10'' E
5. 36'' P: U can be gotten from sea but cost is higher than the market. C: unless cost of getting from sea would decline, then get U from C is commercial feasible
evaluate: whether the cost could decline? choose C ABDE irr.
正确答案 A
作者: baseboss 时间: 2012-5-17 20:36
B:comtemporary firms need to avoid the erosion of power.
P:Society reduce the power when the power is not be used reponsible.
C:in business,firms want to remain power need to act responsible.
推测;
act responsible will gradual erosion of power.
(A) Government institutions are as subject to the
Iron Law of Responsibility as business
institutions.
irrelevant
(B) Public relations programs can cause society to
consider an institution socially responsible
even when it is not.
irrelevant
(C) The power of some institutions erodes more
slowly than the power of others, whether they
are socially responsible or not.
irrelevant
(D) Since no institution is eternal, every business
will eventually fail.
irrelevant
(E) Some businesses that have used power in
socially responsible ways have lost it
right
2.B:USA currently manufacture most of solar-power generater,most of them are export to Euro.Euro also start to manufacture solar-power.
P:USA want to let local people use solar-power.
C: NA
推测:
With largest production level,the cost will be lower than Euro's
选E maintain 是维持不是继续
3 B:Smoker's lung fuction is worse than non-smoker.
P:Vitamin and mineral is good for lung function.
C:smoker eat more vitamin and mineral rich food to improve the lung function
推测:
improve should more than smoke worse.
选D 凭空想象--》文章中没有提到的
4.B commet is in collision with J star.people want to know the size of fragment.
 :Either L or J atomspere has sulfur ,but inner J atomspere has sulfur
C PENETRATE THE J atomspere without burn up
推测:
give p and c
选E
5.B:U can from mines and seawater.
 :From seawater cost more than sale price
c:Only the cost of seawater U reduce,will use a lot
推测:
MINES WILL BE USED OFF.seawater cost will be accept.
选A
作者: 二楼往下掉 时间: 2012-5-28 23:16
1. weaken
(1)1min
(2)逻辑链
Speaker:
Contemporary business firms need to avoid social responsibility leads to the gradual erosion of power
Responsibility:
Society ultimately acts to reduce power when society thinks it is not being used responsibly
Conclusion:
a business that wishes to retain its power as long as it must act responsibly
(3)
(A) Government institutions are as subject to the Iron Law of Responsibility as business institutions.
——原文没提到Government institution
(B) Public relations programs can cause society to consider an institution socially responsible even when it is not.
——原文将social responsibility与power成正比,但B说不一定成正比,所以weaken
(C) The power of some institutions erodes more slowly than the power of others, whether they are socially responsible or not.
——没有比较速度的问题
(D) Since no institution is eternal, every business will eventually fail.
——不知道为啥不对,就是觉得不对。。。
(E) Some businesses that have used power in socially responsible ways have lost it.
——举反例,但是为啥不对??
2.
(1)计时:45s
(2)逻辑链
Background:
<1>United States manufacturers currently produce most of the world's solar-power generators--most of which are exported to Europe.
However
<2> European manufacturers are emerging and probably will ultimately capture much of the European market
<3> The United States government is funding initiatives intended to encourage use of solar power within the United States.
Primes:
<1> if these initiatives succeed in increasing the demand for solar-power generators in the United States
<2>?
Conclusion:
United States manufacturers will probably maintain significant production levels
(3)
(A) some United States manufacturers have been substantially increasing their output over the last several years
——没有提到output的问题,无关
(B) the efficiency of solar-power generators in converting energy from the Sun into electric power is not improving as fast as it once did
——文中没提到efficiency of converting energy,无关
(C) just as European manufacturers enjoy certain competitive advantages in Europe, so do United States manufacturers in the United States
——美国国内市场有很大竞争力,所以可以持续大量生产
(D) European governments are currently undertaking initiatives to stimulate the use of solar power within Europe
——跟美国半毛钱关系没有,无关
(E) the current market for solar-power generators in the United States is very limited
——weaken
3.
(1)计时45s
(2)逻辑链 assumption:取非是否削弱argument
background:
<1>higher level of certain V and minerals in bloodstream —> better lung function
<2>the lung function of smokers <nonsmokers
Plan:
Increase intake of foods that are rich in these V and minerals
Goal:
For smokers to improve lung function
4.这题用的是XDF的方法,觉得还可以
5.
(1)计时:45s
(2)逻辑链 evaluate
Background:
Most of the world's supply of uranium currently comes from mines.
Primes:
It is possible to extract uranium from seawater, but the cost of doing so is greater than the price that uranium fetches on the world market.
Conclusion:
Until the cost of extracting uranium from seawater can somehow be reduced, this method of obtaining uranium is unlikely to be commercially viable.
(3)猜测
<1> whether seawater will be the only way to obtain the uranium?
<2> whether obtain uranium from mines will be more expensive?
Which of the following would it be most useful to determine in evaluating the argument?
作者: emmasy 时间: 2012-6-3 21:16
1.
premise:In the long run, company which avoids social responsibilities will eventually lose its power. This is proved by history.
conclusion:To retain its power, the company have to act responsibly.
a.support
b.weaken
c.irrevelant
d.irrevelant
e.irrevelant
2.
background:E will have much of the market which used to belong to US. So US is encouraging the use of the product within the contry.
premise: this strategy succeed
conclusion:US can maintain its production level which is reduced by E.
a.irrevelant
b.irrevelant
c.support
d.irrevelant
e.weaken
3.
backgroud:The higher the level of certain vitamins and minerals in the bloodstream, the better a person's lung function, as measured by the amount of air the person can expel in one second.
premise:The lung function of smokers is significantly worse, on average, than that of nonsmokers
conclusionne way for smokers to improve their lung function is for them to increase their intake of foods that are rich in these helpful vitamins and minerals
a.irrevelant
b.irrevelant
c.irrevelant
d.irrevelant
e.diminish other possible factor which may affect the conclusion
4.a
5.
backgroud: There are two ways which are used to gain U. However, using seawater is much expensive than using mine.
premise: until the cost of the way which uses seawater can be reduced.
conclusion: this way can be commercially viable.
a.irrevelant
b.irrevelant
c.irrevelant
d.if it is yes, seawater will be used no matter the cost of the method;weaken. if it is no, support
e.irrevelant
作者: emmasy 时间: 2012-6-3 21:19
最后一题的A和D我觉得意思差不多的啊,A是说陆地上的快用完了,D是说海水里的比陆地上的多,为什么选A呢?
求解释啊!是我理解错了么?
作者: jetyxo 时间: 2012-6-5 23:09
啊啊啊,一不小心按错键写的全都木有啦,伤心了,把题做完了,逻辑链就补不了额....
作者: bet 时间: 2012-6-6 23:24
跟楼主请假啊,明早补上!!真不好意思 要断网了。。。。
现在补上
1 1’03”
P:avoiding social responsibility leads to the gradual erosion of power.
C: a business that wishes to retain its power as long as it can must act responsibly.
预测:other factors can still affect the retaining of power
选E 不确定ABCD无关
2 40”
BG: The United States government is funding initiatives intended to encourage use of solar power within the United States.
P: United States manufacturers will probably maintain significant production levels.
预测:gap 美国人都倾向于买本国产品
选C跟预测的一样其他无关
3 29”
P: The higher the level of certain vitamins and minerals in the bloodstream, the better a person's lung function.
C: one way for smokers to improve their lung function is to increase their intake of foods that are rich in vitamins and minerals.
预测:smokers’ level of V and M in bloodstream is lower than normal people.
选E 但是B也觉得对
457”
P:The fragments themselves almost certainly contained no sulfur, but the cloud layer below Jupiter's outer atmosphere does contain sulfur.
Hypothesis P: sulfur would have seeped into the outer atmosphere if comet fragments had penetrated this cloud layer.
C:it is likely that some of the fragments were at least large enough to have passed through Jupiter's outer atmosphere without being burned up.
选E
5 29”
P: It is possible to extract uranium from seawater, but the cost of doing so is greater than the price that uranium fetches on the world market.
C: until the cost of extracting uranium from seawater can somehow be reduced, this method of obtaining uranium is unlikely to be commercially viable.
预测:U的价格会不会升高
选A
作者: penguinsdz 时间: 2012-6-10 06:27
标题: 6.08+【1-4】
6.08+【1-4】
1. Speaker: Contemporary business firms need to recognize that avoiding social responsibility leads to the gradual erosion of power. This is Davis and Blomstrom’s Iron Law of Responsibility: “In the long run, those who do not use power in a manner which society considers responsible will tend to lose it.” The law’s application to human institutions certainly stands confirmed by history. Though the “long run” may require decades or even centuries in some instances, society ultimately acts to reduce power when society thinks it is not being used responsibly. Therefore, a business that wishes to retain its power as long as it can must act responsibly.
Which one of the following statements, if true, most weakens the speaker’s argument?
(A) Government institutions are as subject to the Iron Law of Responsibility as business institutions.
(B) Public relations programs can cause society to consider an institution socially responsible even when it is not.
(C) The power of some institutions erodes more slowly than the power of others, whether they are socially responsible or not.
(D) Since no institution is eternal, every business will eventually fail.
(E) Some businesses that have used power in socially responsible ways have lost it.
Background Information:
Contemporary business firms need to recognize that avoiding social responsibility leads to the gradual erosion of power.
Premise:
Society ultimately acts to reduce power when society thinks it is not being used responsibly.
Conclusion:
A business that wishes to retain its power as long as it can must act responsibly.
推测(Prephrase: mentally formulate your answer to the question stem):
Even if a business has acted responsibly, it can’t retain its power.
选项分析:选E
2. Which of the following most logically completes the argument?
United States manufacturers currently produce most of the world's solar-power generators--most of which are exported to Europe. However, European manufacturers are emerging and probably will ultimately capture much of the European market. The United States government is funding initiatives intended to encourage use of solar power within the United States. If these initiatives succeed in increasing the demand for solar-power generators in the United States, United States manufacturers will probably maintain significant production levels, since __________.
(A) some United States manufacturers have been substantially increasing their output over the last several years
(B) the efficiency of solar-power generators in converting energy from the Sun into electric power is not improving as fast as it once did
(C) just as European manufacturers enjoy certain competitive advantages in Europe, so do United States manufacturers in the United States
(D) European governments are currently undertaking initiatives to stimulate the use of solar power within Europe
(E) the current market for solar-power generators in the United States is very limited
Background Information:
United States manufacturers produce most of the world’s solar-power generators.
Premise:
These initiatives succeed in increasing the demand for solar-power generators in the US.
Conclusion:
United States manufacturers will maintain significant production levels.
推测(Prephrase: mentally formulate your answer to the question stem):
US should fund initiatives to stimulate the use of solar power within US.
选项分析:选C
3. The higher the level of certain vitamins and minerals in the bloodstream, the better a person's lung function, as measured by the amount of air the person can expel in one second. The lung function of smokers is significantly worse, on average, than that of nonsmokers. Clearly, therefore, one way for smokers to improve their lung function is for them to increase their intake of foods that are rich in these helpful vitamins and minerals.
Which of the following is an assumption on which this argument depends?
(A) Smokers are less likely than nonsmokers to have diets that are rich in vitamins and minerals.
(B) The lung function of smokers whose diets are rich in those vitamins and minerals is generally better than that of nonsmokers with comparable diets.
(C) People whose diets are deficient in those vitamins and minerals do not typically have other health problems in addition to diminished lung function.
(D) Stopping smoking will not typically improve lung function more than any diet changes can.
(E) Smoking does not introduce into the body chemicals that prevent the helpful vitamins and minerals from entering the bloodstream.
Background Information:
The higher the level of certain vitamins and minerals in the bloodstream, the better a person’s lung function.
Premise:
The lung function of smokers is significantly worse than that of nonsmokers.
Conclusion:
One way for smokers to improve their lung function is to increase their intake of foods contain these vitamins and minerals.
推测(Prephrase: mentally formulate your answer to the question stem):
Whether the helpful vitamins and minerals are prevent from entering the bloodstream.
选项分析:选E
4. Astronomer: Observations of the Shoemaker-Levi comet on its collision course with Jupiter showed that the comet broke into fragments before entering Jupiter's atmosphere in 1994, but they did not show how big those fragments were. In hopes of gaining some indication of the fragments' size, astronomers studied spectrographic analyses of Jupiter's outer atmosphere. These analyses revealed unprecedented traces of sulfur after the fragments' entry. The fragments themselves almost certainly contained no sulfur, but many astronomers believe that the cloud layer below Jupiter's outer atmosphere does contain sulfur.Since sulfur would have seeped into the outer atmosphere if comet fragments had penetrated this cloud layer, it is likely that some of the fragments were at least large enough to have passed through Jupiter's outer atmosphere without being burned up.
In the astronomer's argument, the two portions in boldface play which of the following roles?
(A) The first presents a circumstance for which the astronomer offers an explanation; the second is part of that explanation.
(B) The first acknowledges a consideration that weighs against the conclusion of the argument; the second is that conclusion.
(C) The first acknowledges a consideration that weighs against the conclusion of the argument; the second provides evidence in support of that conclusion.
(D) The first provides evidence in support of the conclusion of the argument; the second acknowledges a consideration that weighs against that conclusion.
(E) The first is a judgment advanced in support of the conclusion of the argument; the second is that conclusion.
Background Information:
Astronomers studied spectrographic analyses of Jupiter’s outer atmosphere in order to analyze the size of the fragments.
Premise:
Sulfur would have seeped into the outer atmosphere if comet fragments had penetrated this cloud layer.
Conclusion:
Some of the fragments were at least large enough to have passed through Jupiter’s outer atmosphere without being burned up.
选项分析:选E
5. Most of the world's supply of uranium currently comes from mines. It is possible to extract uranium from seawater, but the cost of doing so is greater than the price that uranium fetches on the world market. Therefore, until the cost of extracting uranium from seawater can somehow be reduced, this method of obtaining uranium is unlikely to be commercially viable.
Which of the following would it be most useful to determine in evaluating the argument?
(A) Whether the uranium in deposits on land is rapidly being depleted
(B) Whether most uranium is used near where it is mined
(C) Whether there are any technological advances that show promise of reducing the cost of extracting uranium from seawater
(D) Whether the total amount of uranium in seawater is significantly greater than the total amount of uranium on land
(E) Whether uranium can be extracted from freshwater at a cost similar to the cost of extracting it from seawater
Background Information:
Most of the world’s supply of uranium currently comes from mines.
Premise:
The cost of extracting uranium from seawater is greater than the price that uranium fetches on the market.
Conclusion:
The method is unlikely to be commercially viable unless the cost can be reduced.
推测(Prephrase: mentally formulate your answer to the question stem):
If the uranium in deposits on land is rapidly being depleted, the cost is unnecessary.
选项分析:选A
作者: LuckyYolandaLi 时间: 2012-6-19 22:35
补昨天的~~ps:学校这儿无限网络太不给力,网页打开老费劲了!
1
1)计时:85s
2)逻辑链:
Premise: Contemporary business firms need to recognize that avoiding social responsibility
leads to the gradual erosion of power. “In the long run, those who do not use power in a manner which society considers responsible will tend to lose it.” The law’s application to human institutions certainly stands confirmed by history.
Concession: Though the “long run” may require decades or even centuries in some instances, society ultimately acts to reduce power when society thinks it is not being used responsibly.
Conclusion: A business that wishes to retain its power as long as it can must act responsibly.
3)推测:没想出来。。。
4)选项分析:选C(正确答案B)
A: Irrelevant.
C: Correct.
2
1)计时:59s
2)逻辑链:
Background information: United States manufacturers currently produce most of the world's solar-power generators--most of which are exported to Europe.
Premise: European manufacturers are emerging and probably will ultimately capture much of the European market. The United States government is funding initiatives intended to encourage use of solar power within the United States and these initiatives succeed in increasing the demand for solar-power generators in the United States.
Conclusion: United States manufacturers will probably maintain significant production levels
3)推测:The decreased demand for American solar-power generators in Europe will be made up with the increased demand in the US.
4)选项分析:选C
3
1)计时:53s
2)逻辑链:
Background information: The higher the level of certain vitamins and minerals in the bloodstream, the better a person's lung function.
Premise: The lung function of smokers is significantly worse, on average, than that of nonsmokers.
Conclusion: one way for smokers to improve their lung function is for them to increase their intake of foods that are rich in these helpful vitamins and minerals.
3)推测:The level of certain vitamins and minerals in the bloodstream can be raised by taking in foods that are rich in vitamins and minerals.
4)选项分析:选E
4
1)计时:110s
2)逻辑链:
Background information: Observations of the Shoemaker-Levi comet on its collision course with Jupiter showed that the comet broke into fragments before entering Jupiter's atmosphere in 1994, but they did not show how big those fragments were. In hopes of gaining some indication of the fragments' size, astronomers studied spectrographic analyses of Jupiter's outer atmosphere. These analyses revealed unprecedented traces of sulfur after the fragments' entry.
Premise: The fragments themselves almost certainly contained no sulfur, but many astronomers believe that the cloud layer below Jupiter's outer atmosphere does contain sulfur. Sulfur would have seeped into the outer atmosphere if comet fragments had penetrated this cloud layer
Conclusion: It is likely that some of the fragments were at least large enough to have passed through Jupiter's outer atmosphere without being burned up.
3)推测:Ⅰpremise; Ⅱconclusion.
4)选项分析:选E
5
1)计时:37s
2)逻辑链:
Background information: Most of the world's supply of uranium currently comes from mines.
Premise: It is possible to extract uranium from seawater, but the cost of doing so is greater than the price that uranium fetches on the world market.
Conclusion: until the cost of extracting uranium from seawater can somehow be reduced, this method of obtaining uranium is unlikely to be commercially viable.
3)推测:。。。
4)选项分析:选E(正确答案A)
作者: angelafeng 时间: 2012-6-21 16:38
4.45S
Background information:Contemporary business firms need to recognize that avoiding social responsibility leads to the gradual erosion of power. This is Davis and Blomstrom’s Iron Law of Responsibility: “In the long run, those who do not use power in a manner which society considers responsible will tend to lose it.” The law’s application to human institutions certainly stands confirmed by history.
Premise: Though the “long run” may require decades or even centuries in some instances, society ultimately acts to reduce power when society thinks it is not being used responsibly.
Conclusion:a business that wishes to retain its power as long as it can must act responsibly.
Prephrase: (weaken) sometimes, although the business act responsibly, the public don’t think so.
(A) Government institutions are as subject to the Iron Law of Responsibility as business institutions. -----------the above argument don’t mention government institutions.
(B) Public relations programs can cause society to consider an institution socially responsible even when it is not.------------------correct
(C) The power of some institutions erodes more slowly than the power of others, whether they are socially responsible or not. ----------------------the argument don’t mention any comparison between socially responsible institutions and non-socially responsible institutions.
(D) Since no institution is eternal, every business will eventually fail. ---------------------support the argument
(E) Some businesses that have used power in socially responsible ways have lost it. -------------support the conclusion, the difference between retaining power and wishing to retain power cannot be ignored.
1.46s
Background information: United States manufacturers currently produce most of the world's solar-power generators--most of which are exported to Europe. However, European manufacturers are emerging and probably will ultimately capture much of the European market. The United States government is funding initiatives intended to encourage use of solar power within the United States.
Premise : if these initiatives succeed in increasing the demand for solar-power generators in the United States,
Conclusion : United States manufacturers will probably maintain significant production levels.
Prephrase: (explain)一旦美国本土市场打开,对太阳能的需求比欧洲更大。
Answer:C
1.31s
The higher the level of certain vitamins and minerals in the bloodstream, the better a person's lung function, as measured by the amount of air the person can expel in one second. The lung function of smokers is significantly worse, on average, than that of nonsmokers. Clearly, therefore, one way for smokers to improve their lung function is for them to increase their intake of foods that are rich in these helpful vitamins and minerals.
prephrase:(assumption)假设吸烟不会阻碍营养物质的吸收。
Answer:E
2.56s
Background information:Observations of the Shoemaker-Levi comet on its collision course with Jupiter showed that the comet broke into fragments before entering Jupiter's atmosphere in 1994, but they did not show how big those fragments were. In hopes of gaining some indication of the fragments' size, astronomers studied spectrographic analyses of Jupiter's outer atmosphere. These analyses revealed unprecedented traces of sulfur after the fragments' entry.The fragments themselves almost certainly contained no sulfur, but many astronomers believe that the cloud layer below Jupiter's outer atmosphere does contain sulfur.
Premise:Since sulfur would have seeped into the outer atmosphere if comet fragments had penetrated this cloud layer.
Conclusion:it is likely that some of the fragments were at least large enough to have passed through Jupiter's outer atmosphere without being burned up.
answer:E
4.28S
Background information:Most of the world's supply of uranium currently comes from mines.
Premise:It is possible to extract uranium from seawater, but the cost of doing so is greater than the price that uranium fetches on the world market.
Conclusion: until the cost of extracting uranium from seawater can somehow be reduced, this method of obtaining uranium is unlikely to be commercially viable.
prephrase:(evaluate)是否在能提取uranium的mines不多了。
Answer:A
作者: 呵呵牙 时间: 2012-6-22 11:59
1. 50s
Background Information: Contemporary business firms need to recognize that avoiding social responsibility and use power. In the long run, society considers responsible will tend to lose power if firms do not use it.
Premise: Though the “long run” may require more times, society ultimately acts to reduce power when society thinks it is not being used responsibly.
Conclusion: Business want to retain power as long as it can must act responsibly.
Prephrase: Business can retain power in other way, such as though “long run”.
选E ————————对比答案分析, 这个选项与conclusion陈述有差别
A (government institutions)——irrelevant
B (public relations program)——irrelevant —————————correct
C (power of institutions) ——irrelevant
D (every business will fail)——irrelevant
2. 37s
Background Information: US manufacturers produce most of the world’s solar-power generators—most of which are exported to Europe, but European manufacturers capture much of the European market. The US government encourages use of solar power within the US
Premise: These initiatives succeed in increasing the demand for solar-power generators in the US.
Conclusion: US will probably maintain significant production level.
Prephrase: European would like use their native things, so the US government want to American just use their own solar-power generators.
选C
3. 39s
Background Information: The higher level vitamins and minerals are better to people’s lung function.
Premise: The lung function of smokers is worse than nonsmokers.
Conclusion: One way for smokers to improve their lung function is to increase their intake vitamins and minerals.
Prephrase: Smokers can continue smoke and at the same time intake more vitamins and minerals.
选E
4. 50s
Premise: sulfur would have seeped into the outer atmosphere if comet fragments had penetrated this cloud layer
Conclusion: some of the fragments were at least large enough to have passed through Jupiter's outer atmosphere without being burned up.
其实这道题最后一句是开头since表示解释,后面一定跟的是结论. 因为段落中没有明显转折,所以第一个黑体句是support的.
选E
5. 32s
Premise: Uranium is possible from seawater, but it cost high price.
Conclusion: Until the cost of extracting uranium from seawater can be reduced, this way is unlikely to be commercially viable.
选A
作者: jiajiajudy 时间: 2012-6-23 06:25
1.
Avoiding social responsibility? lost power
Society reduce power when they think it is not? retain power as long as it can must act responsibly
(B) Public relations programs can cause society to
consider an institution socially responsible
even when it is not.
2.
BG:US SPG most are expoted to Europe
Europe produce themselves capture E market? US encourage use of spg in US-?matain production levels
C) just as European manufacturers enjoy certain competitive advantages in Europe, so do United States manufacturers in the United States
3.
BG:Higher level of vitamins and minerals means better lung function(air expel per second)
Smoker is worse than nonsmoker? increase intake of foods
(B) The lung function of smokers whose diets are rich in those vitamins and minerals is generally better than that of nonsmokers with comparable diets.
4.
BG:comet broke before entering Jupiter , but don’t know how big these fragments are
Study Jupiter’s outer atmosphere?史无前例 有sulfur after fragments ‘entry ?cloud below outer atmosphere 有sulfur
因为fragments有sulfur 让没有sulfur 的outer atmosphere 有sulfur? fragments 非常大
(E) The first is a judgment advanced in support of the conclusion of the argument; the second is that conclusion.
5.
from weasater 成本高于售价? uranium comes from mines? 除非成本降低,否则就不经济
(A) Whether the uranium in deposits on land is rapidly being depleted
作者: cleotina 时间: 2012-6-27 13:20
在写字板写的,悲剧没存
简单总结一下:
1 做错了 读题用了两分钟 还是不明白
4 Boldface 在两个选项间犹豫,最终选错了。写逻辑练得时候太简化。
其他三道读完题准确预测出答案
作者: Donts 时间: 2012-7-14 10:33
1.计时1'21''
premise:the society will act to reduce power when it thinks someone to be not responsible.
conclusion: if a business wanna stay in power, it must act responsibly.
推测:to attack a conditional conclusion, 需要证明act responsibly不是唯一方法to stay in power.
(A) Government institutions are as subject to the
Iron Law of Responsibility as business
institutions.关GI什么事呢,没有attack到conditional conclusion.
(B) Public relations programs can cause society to
consider an institution socially responsible
even when it is not.
(C) The power of some institutions erodes more
slowly than the power of others, whether they
are socially responsible or not.erode的快慢与破解act responsibly无关
(D) Since no institution is eternal, every business
will eventually fail. attack了if
(E) Some businesses that have used power in
socially responsible ways have lost it.
选择B
没有时间重复看逻辑链里的prep了TAT,都是做过的题目,都有印象,影响练习,都作精炼题直到逻辑链都是不认识的。
很开心做了carpet的题目对attack conditional conclusion有了印象,并且运用起来还做对了,但是读题速度很让我泪奔。
问题:conditional conclusion是不是都去attack then呢?plan-goal的题目都是去attack plan吗?
作者: showysimple 时间: 2012-7-20 14:45
7.19+[1-4]
1.
[Premise] avoiding social responsibility --> gradual erosion of power; society ultimately
acts to reduce power when society thinks it is not being used responsibly
[Conclusion] business that wishes to retain its power as long as it can must act responsibly.
[Keyword] 缺乏社会责任但不失power
[Pre-phrase] 公众不认为该公司缺乏社会责任
A - non-relevant
B - right answer
C - non-relevant
D - out of scope
E -
2.
[Premise]
- U.S. most of SP generators are exported to Europe market where local manufacturers are predominant;
- U.S. manufacturers product most of the SP generators;
- U.S. gov. encourage local demand
[Conclusion] U.S. manufacturers will prob. maintain significant production levels
A - 'output in the past' is irrelevant
B - 'efficiency of generator' is irrelevant
C - right answer (美国市场扩大后,美国制造商未必是赢家,除非受本国的竞争保护,像欧洲市场那样。)
D - 结论涉及的是'U.S. market', not 'Euro market'
E - irrelevant
3.
[Premise] 血液含维他命和矿物成分越高,肺功能越好。吸烟者肺功能不好。
[Conclusion] 吸烟者多吃富含维他命和矿物成份的食物有利于提高肺功能。
A - wrong。“吸烟者很少吃富含维他命和矿物成份的食物”与结论相悖
B - ?
C - irrelevant
D - 不直接。“戒烟不比改善饮食对肺功能起效”不代表就应该选择“改善饮食”
E - right answer
作者: TICKCOCK 时间: 2012-8-3 23:55
1、Background: Iron Law of Responsibility states that in the long run, company who do not use power in a manner which society considers responsible will tend to lose it.
Premise: though “long run” means a long period. The society would reduce the power only when it found careless manner.
Conclude: the corporation should always behave in a manner way.
Prephase: behave bad occasionally may be OK
Answer: B
2、US manufacturer produce most of solar-power generator. And most of generator are exported to Europe. However, the EU country company would compete with US manufacturer about EU market soon.
Conclusion: one funding initiatives for US solar power within US would work for US manufacturer about maintaining the present production level since^^^
Answer E(C)
3、background: smoker’s lung function is worse than that of nonsmoker on average.
Premise: the more vitamin and mineral in the bloodstream, the better the lung function is, measured as the amount of air the person can expel in a second.
Conclusion: it is beneficial for nonsmoker to intake food full of vitamin and mineral.
Answer: E
4. astronaut learn the spectrographic analyses of Jupiter’s outer atmosphere to know the size of fragment. Surprisingly find trace of sulfur.
Since sulfur exist in cloud layer rather than fragments. The fragments would have speeded into atmosphere. It shows the big size of commet.
Answer: E
5: most of uranium currently come from mine. Extract Uranium from sea is expensive.
Premise: only if the cost of extract uranium from seawater could be reduced.
Conclusion: the method that get uranium from sea is valuable.
Answer: A
作者: wanggang0411 时间: 2012-8-9 14:27
1 40
Conclusion: If a business wishes to retain its power, it must act responsibly.
Sufficient: retrain power
Necessary: act responsibly
Inference: Weaken the necessary to testify that the necessary condition may not occur for the required sufficient condition.
2 42
Premise: most SG of US exported to European market.
Premise: European SG manufactures are emerging in European market.
Premise: US government will initiative US domestic market to maintain the production of US’s manufacture.
Conclusion: The production level will maintain significantly since_
Inference: A. the products of European manufacture rarely exports to US.
B. US’s customers are only interesting on domestic product.
3 30
Premise: V and M are good for person’s lung.
Premise Smoker’s lung is less functional than that of Non-smoker.
Conclusion: a way to improve the function of smoker’s lung is to increase their V and M ingestion.
Assumption: Smoking doesn’t impact the intake of V and M.
4 1:04
For this question:
I think the last sentence is the conclusion of the argument.
And the first sentence is a premise of the argument to help deduce the conclusion.
The sentence between first one and second one is this reasoning.
5 19
Premise: one way to get URN is from mines;
Premise: another way to get URN is from seawater;
Premise: Price of URN from seawater is greater than that from mines;
Conclusion: Until the cost to extract URN from seawater reduced, this method is not commercial viable.
Answer A: if the deposits in earth will be depleted, then the method of seawater will be viable.
Reversely, the method will not be viable.
作者: FannyL 时间: 2012-8-17 13:52
1.50'Weaken
读起来感觉就有点绕。。
background:business firms need to recognize avoiding social responsibility lead to the gradual erosion of power
premise:if you want power,you must use responsibly well.
conclusion:a business wishes to retain its power if it can act responsibly
the relationship between the responsibly and power 难道真的是正比关系?
A:support
Bublic relations programs is irrevelent
C:不能充分说明
D:同C
E:right
2.38'
background:USA export most of the solar-power to Europe.
premise:USA try to encourage use of solar power within the USA
conclusion:USA manufaccturers will significant production levels
选E
3.33’
background:the higher the level of certain vitamins and minerals in the bloodstream,the better a person's lung function
premise:smokers' lung function is worse
conclusion:smokers increase lung function by eat foods that are rich in these helpful vitamins and minerals
A:无依据
B:对比不成立
C:无依据
D:无依据
E:right
4.55’
选D 第二个不像是结论。。好吧 每次做这种题都不行
5.40’
Evaluate
background:most of the world's supply of uranium come from mines,and the cost of extracting uranium from seawater is expensive
premie:the cost of extracting uranium from seawater can be reduced
conclusion:the method of obtainning uranium is wildely used
D (没感觉 一直在AD徘徊。。)
作者: chengzaaaa 时间: 2012-8-21 21:41
4.
1)43”weaken
2)BI: this law: long run, have power and not use, will lose the power.
P: the law is very applicable to the society.
C: business gotta act responsibly if it wants to retain its power.
3)besides act responsibly, there are other ways to retain the power.
4)E B
5)A. Range. We are discussing business firms, not gov institutions.
B. not weakening…
C. irrelevant comparison
D. does not make sense
E. Correct. Not very sure……….
1.
1)34”complete
2)BI: US productions go to Europe. E productions are going up and taking the E market. US making effort to increase local demand.
P: the increasing-demand program goes well and _________
C: US production levels are doing great.
3)US put a lot of effort marketing the thing in US.
C
2.
1)23”assumption
2)BI: lung function, measured by air expel amount per sec. smokers’ lung, worse than nonsmokers.
P: higher v and m, better lungs.
C: smokers. Improve lung. Eat food rich in v and m.
3)damaged lungs do not interfere with the absorbing of the v and m.
E
3.
1)59”boldface
2)BI:
P:
C:
3)1st. evidence, findings.
2nd. Conclusion based on the findings.
A E
4.
1)23”evaluate
2)BI: U comes from mines. From sea is ok but pricy.
P: cost of sea-tracting is higher than market price
C: unless cost of sea-tracting is reduced, this method is not commercially good.
3)whether there is more U in the mine for people to extract.
A
谢谢lz!!!
作者: Maykey 时间: 2012-8-26 17:34
BG: business firm avoid social responsibility may lead to the erosion of power
Those do not use power that consider responsible will lose it
The implication of law confirmed by history
Premise: society will reduce power that consider irresponsible
Conclusion: business must act responsible if they want to retain their powers
推测:apart from act responsible, some other factor need to be followed by business, in order to retain power
A. 无关
B. 无关
C. 无关
D. 无关
E.correct. Not all the business that act responsible will retain its power
1.
38S
BG:US manufacture most of the solr power generators,most export to Europe
Europen mnufactuer emerge and will gain much of Europe market share
Premise: US government fund intiactive to encourage use of Solar P in US
Conclusion:If the intiactives succeed, US matian its production levels
推测:US has competivive advantages in US local market
A. In this case,Us will decrese it s production levels
B. 无关
C. Correct
D. 无关
E. Shell game, if it is limited, the us govern will not need to fund intiactive
作者: 我心匪席 时间: 2012-8-29 21:50
1) Time: 45’
2) Background Information: Business communities will lose society power if they avoid using it.
Premises: Business wishes to retain its power.
Conclusion: Business should act responsibly.
3)  rephrase: Other ways to retain its power
4) A-----irrelevant
B-----right
C-----irrelevant
D-----irrelevant
E------irrelevant
个人觉得这是个充分必要条件的weaken: 背景是WRP的发生一定要AR,即AR是WRP的必要条件(necessary),需要weaken, 则可以论证AR时充分条件(sufficient)就行了,所以答案B就给出了这一weaken,即不必要,充分就行。
====================================
1) Time: 28’
2) Background Information: US experts SG to Euro. However, Euro. has its own SG. US makes some initiatives in US to increase national demand for SG.
Premises: Initiatives in US is successful.
Conclusion: US maintain significant production levels.
3)  rephrase: US has advantage in SC
4) A-----irrelevant
B-----irrelevant
C-----right
D-----irrelevant
E------weaken
=====================================
1) Time: 30’
2)  remises: Smokers to improve lung functions
Conclusion: Increase their intake of foods rich in vitamins and minerals
3)  rephrase: Quit smoking, attack necessary
4) A-----irrelevant
B-----irrelevant
C-----irrelevant
D-----irrelevant
E------right
======================================
1) Time: 52’
2)  remises: B1
Conclusion: B2
E------right
======================================
1) Time: 22’
2) Background Information: Most of the world's supply of uranium currently comes from mines. It is possible to extract uranium from seawater, but the cost of doing so is greater than the price that uranium fetches on the world market.
Premises: until the cost of extracting uranium from seawater can somehow be reduced
Conclusion: this method of obtaining uranium is unlikely to be commercially viable.
3)  rephrase: Other methods
4) A-----right (攻击必要条件,提供充分条件)
B-----irrelevant
C-----irrelevant
D-----irrelevant
E------irrelevant
作者: srafcatt 时间: 2012-10-5 14:48
标题: 1-4
精炼46s weaken
演讲者:现代商业公司需要认识到避开社会责任会导致权力的渐次侵蚀。这是Davis和Blomstrom的责任铁律:"从长远来看,那些不以一种让社会觉得有责任感的方式行使权力的人最终会失去权力。"这个规律对于人类机构的应用在历史上的确是站得住脚的。虽然“长远”某种程度上可能需要几十年甚至上百年,社会最终行动起来去削减权力,当社会认为权力没有被有责任感的使用。因此,一个企业若期待它的权力尽可能长久就必须履行责任。
BG:in the long run---power will lose--if not use it responsible as the society thinks
premise:retain its power as long as it wishes
conclusion:business act responsibility
if the responsible business lose because of responsibility when competing with competitors,the logic is wrong.
choice:B
A irrelevant information,the question is talking about responsibility&power.
B correct
C incorrect.The irresponsible ones exist may due to the not long enough time.
D irrelevant information
E retain its power as long as it wishes is ignored.
逻辑链
1.36s FILL
美国制造商目前生产了世界上大多数的太阳能发电机--这些发电机的大部分出口欧洲。然而欧洲制造商正在产生并很可能将最终占领大量欧洲市场。美国政府开始投资意在鼓励国内使用太阳能的方案。如果这些方案成功地增加了太阳能在国内的需求,美国将很可能保持显著的生产水平,因为____
BG:Europe manufacturers---emerging---solar-power generators'quantity exported to Europe decline
premise:initiatives --succeed to increase the domestic demand
Conclusion:U.S. maintains production level
because solar-power is not wide-spread used in U.S.
choice:E
A irrelevant information The question is talking about why initiatives lead to high production level.
B irrelevant information If the solar power is developing fast,there is no need to discuss if this initiatives will be affectable.
C incorrect,the situation in Europe is good do not necessarily mean it will happen as the same in U.S.
D the same reason as choice C
E correct,because the market is limited,so it is easy to succeed in increasing the use of solar-power.
这个是怎么回事?选C
2.36s assumption
BG:certain vitamin in blood is high, the lung function is better
premise:smokers eat food containing rich certain vitamin
conclusion:their lung function will be better
assumption: only if the vitamin quantity in blood will significantly increase after digesting.
choice:E
A weaken the argument
B This is reasonable,but not necessarily,because the question's goal is to be better,not being better than nonsmokers.
C irrelevant information,the question is talking about vitamin&lung function.
D irrelevant information,the reason is the same as choice C
E correct,this assumption ensures that vitamin will effect the lungs smoothly.
取非后削弱结论,我理解的是命题与逆否命题同真假,因此,assumption类型的题目可以通过判断逆否命题来判断命题的真假。
3.76s boldface
BG:certain comets--fragments--crush the Jupiter
premise:fragments--no sulfur,Jupiter outer space--has sulfur,and if fragments penetrate outerspace,sulfur will be taken
conclusion:fragments'size is large enough to pass through outer space without being burned up.
choice:E
A the first is the support idea for the second part,the second part is the conclusion
B it is support,not weigh against
C it is support,not weigh against
D it's support
E correct
本题的总结:背景信息和支持结论的信息有时是有些相像的。 但是背景信息无法推出结论,而支持性信息可以。因此粗体字的第一部分不是背景,而是支持粗体字第二部分的。
4.38s EVALUATE THE ARGUMENT
premise:the cost of uranium from seawater reduced
conclusion:this method is commercially viable
uranium from mine is vast
chlice:A
A This is useful
B irrelevant,nothing to do with the topic
C The topic is cost decline---->seawater method commercially viable,it is not related to technology for it does not necessarily mean the cost will decline.
D irrelevant,the question is about cost and commercial viable,it has nothing to do with the total amount.
E special case.This is not convincing.
作者: srafcatt 时间: 2012-10-5 15:43
逻辑链第一题:E选项应该直接理解——因为美国国内市场是有限的,所以这项计划不可能成功。是weaken,这里强调了一个思路是直接理解选项的意思,不随便猜测后面的意图。
B选项的意思是欧洲采用好使,所以美国也可能好使,相比之下较其他选项更加可信。
下面这道题来自OG:
The spacing of the four holes on a fragment of a bone flute excavated at a Neanderthal campsite is just what is required to play the third through sixth notes of the diatonic scale—the seven-note musical scale used in much of Western music since the Renaissance. Musicologists therefore hypothesize that the diatonic musical scale was developed and used thousands of years before it was adopted by Western musicians.
Which of the following, if true, most strongly supports the hypothesis?
(E) The cave-bear leg bone used to make the Neanderthal flute would have been long enough to make a flute capable of playing a complete diatonic scale.
(B) No musical instrument that is known to have used a diatonic scale is of an earlier date than the flute found at the Neanderthal campsite.(OG的解释:This also weakens the hypothesis, because it states that there is no known evidence of a certain type that would support the hypothesis.)
从这个解释可以推出OG认为No musical instrument that is known应该直接去理解而不是间接的去推测,即应该直接理解is very limited.所以美国太阳能那题E错在起了weaken的作用
作者: ariel0923 时间: 2012-10-14 15:04
Speaker:avoiding social responsibilityleads to the gradual erosion of power. Therefore, abusiness that wishes to retain its power as longas it can must act responsibly.
(A) Government institutions are as subject to the Iron Law of Responsibility as businessinstitutions.---------irrelevant
(B) Public relations programs can cause society to consider an institution socially responsible even when it is not.-------weaken
(C) The power of some institutions erodes more slowly than the power of others, whether they are socially responsible or not.-------irrelevant
(D) Since no institution is eternal, every business will eventually fail.--------irrelevant
(E) Some businesses that have used power in socially responsible ways have lost it.-------some not all and in this topic the author say"as long as",not enough
作者: mirare 时间: 2012-10-21 12:38
Background:
Avoiding social responsibility leads to the erosion of prower.
Premise:
Citing a sentence from law of responsibility and has been proved by history.
Society reduce power when society thinks it is not being used responsibly in the long run.
Conclusion
Business act responsibly to retain long power.
Prophase:
C
prophase,想不出来~ 但其实就是要反驳说不负责也可以retain long power。
作者: hanhan1991 时间: 2012-11-15 18:03
1.
1)计时:80s(too long)
2)逻辑链:
Background:Avoiding social responsibility often leads to the gradual erosion of power of comtemporate firms. This theory is conformed by history.
Premise:Society ultimately acts to reduce power when society thinks it is not used responsibly although the long run may require a very long period of the time.
Conclusion:A business must act responsibly to retain its power as long as it can.
3)推测:If the business change the kind of power it gains frequently, it may not care about whether one type of the power it has used will lose in future.
****the conclusion is conditional, so we should find answer that states even if the business do not act responsibly, it can retain its power.
4)选项分析:choose B.
A.irrelevant? not very sure.
***we onlu concern about business institute.
B.correct. if this is true, even if the business acts resposibly, it will still loss its power.and vice versa.
C.the speed is irrelevant.
****some is not representative.
D.irrelevant. the argument only concerns about the possible maximal time but not eternal time.
E. some is not representative.
总结:对于conditional conclusion, 要关注由什么到什么,attacking the Mistaken Reversal of the conclusion does not hurt the conclusion
2.
1)计时:44s
2)逻辑链:
Background:American often exported solar powder generators to Europe in the past. However the Europe market will lost since the European manufacture emerging.
Conclusion:US manufacture will probably maintain production level if the intiatives intented to encourage the use of solar powder in US succeeds.
3)推测:Because the US manufacture will definetely lose their market in Europe so if they want to maintain the production level only by increasing their domesticate market. Thus the intiative must help increase people's use of solar power. So maybe before the intiatives solar power did not been widly used in US.
4)选项分析:choose C.
A.some is not representative.
B.this weakens the argument and can not be the reason.
C.correct. if the US manufacture gain more market within the nation, the increasing domestic market will offset their lossing in the Europe.
D.irrelevant. how Europe do is irrelevant.
E.this can be a contender, but no better than C, which directly refers to the manufacturer.
3.
1)计时:44s.
2)逻辑链:
Premise:The higher the level of vitamins and minerals in the bloodstreams the better a person's lung function.
Conclusion:One way to improve the smoker's function is to let them intake more foods that are rich in vitamins and minerals.
3)推测:If intaking more vitamins and minerals can really increase the level of such substance in the bloodstream, this method is feasible.
4)选项分析:choose E
A.irrelevant.
B.this oppose the facts
C.irrelevant.
D.irrelevant. .
E.correct. this rules out the possibility that may prevent the vitamins and minerals from entering the bloodstream.
4.
1)计时:84s.(too long)
2)逻辑链:
Background:Observations showed that the comet broke into fragments before entering Jupiter's atmosphere but they did not show how big those fragments were. Some analyses revealed unprecedented traces of sulfur after the fragments' entry.
Premise:Sulfur would have seeped into the outer atmosphere if comet fragments had penetrated this cloud layer
Assumption:The fragments themselves almost certainly contained no sulfur, but many astronomers believe that the cloud layer below Jupiter's outer atmosphere does contain sulfur.
Conclusion:some of the fragments were at least large enough to have passed through Jupiter's outer atmosphere without being burned up.
3)推测:the first is the assuption of the astronomers, if true helps lead to the conclusion and the second setences is the main conclusion of the argument .
4)选项分析:choose E
A.both the first and the second are wrong.
B.the first is wrong.
C.both the first and the second are wrong.
D.the second is wrong .
E.correct.
5.
1)计时:32s.
2)逻辑链:
Premise:Uranium can be extracted from sweater but the cost of doging so is expensive.
Conclusion:Only if the cost of doing so can be somehow reduced, this method of obtaining uranum is likely to be available.
3)推测:if extracting from sweater can get much more uranium, so the method can still be feasible.
4)选项分析:choose A
A.correct. if this is ture, we would extracting uranium from sweater no matter how expensive it is.
B.irrelevant.
C.this can not strengthen or weaken the argument.
D.irrelevant..
E.irrelevant.
作者: CD用户825193 时间: 2012-11-17 16:52
标题: Daily CR-4_2012-11-17
1. 50" +35" +... (Weak) [Read several times...]
P: Firms avoid acting responsibly will have erision of power in a long run.
C: SFirms which want have power as long as they can must act responsibly.
Pre: No clue.
Answers:
(A) Nothing about socially responsibility, irrelevant
(B) CORRECT
(C) Irrelevant
(D) Irrelevant
(E) Irrelevant
2. " (Blank / Support)
P: US's solar power generators aim European market
P: European's generator manufacturers emerging, will take the European market
P: US government encourage the home generator market.
C: SIf home market is getting bigger -> more customers in US market -> ? -> same production level
Pre:US producers lost EU market but get US market, no overseas like EU producer competitors
Answers:
(A) Increased output of some manufactors last year, irrelevant
(B) Effeciency is not improve as usual -> less consumers -> lower production level: Weak
(C) CORRECT: Certain competitive advantage = capture the home market -> more consumers in US -> same production level
(D) What happens in Europe is irrelevant
(E) Current market is irrelevant
3. 36" (Support)
P: Higher level of vitamin and mineral in bloodstream = better function of lung
P: Smokers lung function is worse than nonsmokers
C: Smokers take more food with vitamin and mineral -> ? -> better lung function
Pre: Vitamin and mineral in food can turn to be the ones in blooestream effectively.
Answers:
(A) Smokers' diet habbit is irrelevant.
(B) Wrong comparision, should compare smokers who have diet rich of vitamin and mineral ans smokers have diet with not.
(C) Other health problems are irrelevant.
(D) Wrong comparision, stopping smoking vs. diet doesn't help.
(E) CORRECT.Smoking doesn't stop the transfer of V&M from food to blood -> higher V&M in bloodstream -> better lung function
4. 58" +... (Function of the BOLD PARTS) [Read a second time to better understand it.]
P: One comet broke into fragments when it collaped into J, and astronomers try to figure out how big the fragments are.
P: There's sulfur on the track comet going to the J in the outer atmosphere
Bold-1: Fragments have no sulfur, but outer atmosphere has.
Bold-2: As the sulfur can get into the fragments to make them burn, the fragments's size can be boncluded as the minimum.
C:
Pre: 1: premise of the conclusion, 2: another premise and the conclusion
Answers:
(A)
(B)
(C)
(D)
(E) CORRECT.
5. 26" (Support)
P: The common way to get U is from mine
P: U can also be extracted from seawater, but the cost is higher than the market value of U.
C: Only if the cost of extracting U from seawater is lower -> extracting U from seawater is practical
Pre: The market price of U is table, will not be higher than the cost.
No other cheaper way to extract U from seawater
Answers:
(A) Weak
(B) Irrelevant
(C) CORRECT
(D) Amout of U in seawater is irrelevant
(E) Freshwater is irrelevant.
作者: CD用户825193 时间: 2012-11-17 18:33
今天的逻辑题目都做对了,有点小开心,但是速度和理解力还是问题,要坚持啊~
作者: 刘腿腿eva 时间: 2012-11-23 14:42
都好认真啊。。
作者: kudoucliff 时间: 2012-12-18 01:45
Truth: Institutions doesn’t use responsible power gradually lose it.
Argument: A business that wishes to retain its power must act responsibly.
Weaken: Some affairs can make business retain that power even though it doesn’t act responsibly.
4. Speaker: Contemporary business firms need to
recognize that avoiding social responsibility
leads to the gradual erosion of power. This is
Davis and Blomstrom’s Iron Law of
Responsibility: “In the long run, those who do
not use power in a manner which society
considers responsible will tend to lose it.” The
law’s application to human institutions
certainly stands confirmed by history. Though
the “long run” may require decades or even
centuries in some instances, society ultimately
acts to reduce power when society thinks it is
not being used responsibly. Therefore, a
business that wishes to retain its power as long
as it can must act responsibly.
Which one of the following statements, if true, most
weakens the speaker’s argument?
(A) Government institutions are as subject to the
Iron Law of Responsibility as business
institutions.
Whether government institutions are subject to the Iron Law is irrelevant to business institutions.
(B) Public relations programs can cause society to
consider an institution socially responsible
even when it is not.
Correct answer, for it provides a possibility that institutions can be felt responsible without acting responsibly
(C) The power of some institutions erodes more
slowly than the power of others, whether they
are socially responsible or not.
The quickness of power erosion has no bearing on the author’s argument.
(D) Since no institution is eternal, every business
will eventually fail.
Irrelevant
(E) Some businesses that have used power in
socially responsible ways have lost it.
Irrelevant
BG: US manufacturers export A to Europe. However, European competitors may capture the shares belonging to US manufacturers now.
Premise: US demands for A increased, US manufacturers will benefit by maintaining significant product level.
Underlying reason: US A buyers prefer US products.
Truth 1: Certain vitamins and minerals in blood make a person’s lung function well.
Truth 2: Smokers don’t have lung function as well as that of nonsmokers.
Premise: Eating foods rich in certain vitamins and minerals can help smokers’ lung function better.
Assumption: Nutrients in such foods can be absorbed into blood.
Background: Comet S broke into fragments before entering J, but researches didn’t show how big the fragments were.
New finding: Trace of sulfur was found after fragments’ entry.
Common sense: Fragments don’t contain sulfur.
Premise: The cloud layer does.
Conclusion: Fragments penetrate atmosphere without burning up.
Background: U can be extracted from both M and S. Now is common in M for in S is expensive.
Premise: Cost of extracting U in S doesn’t lower down.
Argument: Extracting U in S is not commercially viable.
Evaluate: Whether the volume of extracting U in M will be large enough or sustainable.
作者: annieliu830 时间: 2012-12-25 01:21
2012年12月24日
1.
1)逻辑链
原因:长期看来,权力不使用就会失去
结论:如果一个企业想有效维持他的权力,他就必须有责任的去行动
2)推测
削弱
3)选项分析
A无关
B正确,即使没履行责任,却被社会认为履行了责任
C与速度无关
D无关,根本没讨论到履行责任
E无关
2.
1)逻辑链
类比题
因为:欧洲制造商迅速崛起,抓住了大部分欧洲市场。
所以:如果美国政府在美国大量生产,美国制造商就能继续保持较高的生产level
2)推测
加强两者相似的结论
已经告诉我结论是一样的了,那就是类比说前提一样囖
如同欧洲制造商更能抓住欧洲的市场一样,美国制造商也更能抓住美国的市场
3)选项分析
A无关
B削弱
C正确
D即为题中条件
E确实能加强结论,说明美国制造商能够维持较高的生产level,因为市场目前还没有怎么被开发的。但是于题目中的类比无关,因而不选。
3.
1)逻辑链
原因:在血液中维生素和矿物质成分越高,使得一个人的肺功能越好。
结论:要提高吸烟者的肺功能,就要使他们食物中的维生素和矿物质更加丰富。
2)推测
假设,其实也就是能加强结论……
从食物中吃到的维生素和矿物质能被人吸收进入血液中,补充gap
3)选项分析
A无关,与我要想提高吸烟者的肺功能这一目的无关
B这里的比较条件相同的是维生素和矿物质,而题目中的变量是维生素和矿物质,不符合要求
C有没有其他毛病无关
D无关
E正确
4.
2)推测
第一句证据
第二句结论,接在since的后半句,表所以,因而是结论
3)E
5
1)逻辑链
原因:从海水里提炼U的成本太高,比从矿里开采买的价格高太多
结论:在成本降低之前,这种获取U的方式经济上都不可行的
2)推测
Evaluation
市场上开采的U是不是很快就卖完了,没得卖了
3)选项分析
A正确,被开采完,没办法,即使成本高也得从海水里提炼了
B无关,这里比较的是开采成本,并不包括运输成本
C与结论的前提相矛盾,结论的前提是在成本降低之前,这里讨论的是如果成本降低了
D无关,数量多少无关,关键是成本
E无关
作者: zxppx 时间: 2013-1-8 21:10
4. Speaker: Contemporary business firms need to
recognize that avoiding social responsibility
leads to the gradual erosion of power. This is
Davis and Blomstrom’s Iron Law of
Responsibility: “In the long run, those who do
not use power in a manner which society
considers responsible will tend to lose it.” The
law’s application to human institutions
certainly stands confirmed by history. Though
the “long run” may require decades or even
centuries in some instances, society ultimately
acts to reduce power when society thinks it is
not being used responsibly. Therefore, a
business that wishes to retain its power as long
as it can must act responsibly.
Which one of the following statements, if true, most
weakens the speaker’s argument?
(A) Government institutions are as subject to the
Iron Law of Responsibility as business
institutions.
It strengthens the argument.
(B) Public relations programs can cause society to
consider an institution socially responsible
even when it is not.
It fails to talk about the relationship between the power and responsibility.
But please notice the words in the argument that ‘society considers responsible’. So it reveals that if the public relations programs can cause society to consider an institution socially responsible, the programs can retain power, regardless it is really responsible.
(C) The power of some institutions erodes more
slowly than the power of others, whether they
are socially responsible or not.
It seems attractive. But there is no relationship with the speed about erosion. Besides, even though the speed is slower for some institutions, they are still losing their power, rather than retaining it.
(D) Since no institution is eternal, every business
will eventually fail.
It seems attractive, too. I think it is the best answer.
The conclusion permits that the business lose its power and eventually fail, so the answer is not right. Besides, it does not prove that if the institution is not responsible, the institution can still retain its power.
(E) Some businesses that have used power in
socially responsible ways have lost it.
Although it seems very attractive, it fails to prove that responsibility is not the necessary condition to the power, because it only proves that responsibility is not the sufficient condition.
1.
prep 07-13.
Which of the following most logically completes the argument?
United States manufacturers currently produce most of the world's solar-power generators--most of which are exported to Europe. However, European manufacturers are emerging and probably will ultimately capture much of the European market. The United States government is funding initiatives intended to encourage use of solar power within the United States. If these initiatives succeed in increasing the demand for solar-power generators in the United States, United States manufacturers will probably maintain significant production levels, since __________.
(A) some United States manufacturers have been substantially increasing their output over the last several years
Firstly, some manufactures are unrepresentative. Secondly, it fails to prove the relationship between the increase in the demand and production levels.
(B) the efficiency of solar-power generators in converting energy from the Sun into electric power is not improving as fast as it once did
Actually, it seems to weaken the argument, rather than strengthen.
(C) just as European manufacturers enjoy certain competitive advantages in Europe, so do United States manufacturers in the United States
The comparison succeeds to predict that the US manufacturers will experience the similar situation just as the European manufacturers did. So it is the best answer.
(D) European governments are currently undertaking initiatives to stimulate the use of solar power within Europe
But we do not know the condition in US.
(E) the current market for solar-power generators in the United States is very limited
But we do not know how about the further.
2.
prep 07-14.
The higher the level of certain vitamins and minerals in the bloodstream, the better a person's lung function, as measured by the amount of air the person can expel in one second. The lung function of smokers is significantly worse, on average, than that of nonsmokers. Clearly, therefore, one way for smokers to improve their lung function is for them to increase their intake of foods that are rich in these helpful vitamins and minerals.
Which of the following is an assumption on which this argument depends?
(A) Smokers are less likely than nonsmokers to have diets that are rich in vitamins and minerals.
Even though the fact is true, it cannot be the assumption.
(B) The lung function of smokers whose diets are rich in those vitamins and minerals is generally better than that of nonsmokers with comparable diets.
The same to A.
(C) People whose diets are deficient in those vitamins and minerals do not typically have other health problems in addition to diminished lung function.
The same to A.
(D) Stopping smoking will not typically improve lung function more than any diet changes can.
The same to A.
(E) Smoking does not introduce into the body chemicals that prevent the helpful vitamins and minerals from entering the bloodstream.
If smoking does introduce in the body chemicals that prevent the vitamins and minerals from entering the bloodstream, the argument cannot be reasonable. We must find something between the foods that are rich in these helpful vitamins and minerals and the level of certain vitamins and minerals in the bloodstream. So it is the best answer.
3.
prep 07-15.
Astronomer: Observations of the Shoemaker-Levi comet on its collision course with Jupiter showed that the comet broke into fragments before entering Jupiter's atmosphere in 1994, but they did not show how big those fragments were. In hopes of gaining some indication of the fragments' size, astronomers studied spectrographic analyses of Jupiter's outer atmosphere. These analyses revealed unprecedented traces of sulfur after the fragments' entry. The fragments themselves almost certainly contained no sulfur, but many astronomers believe that the cloud layer below Jupiter's outer atmosphere does contain sulfur. Since sulfur would have seeped into the outer atmosphere if comet fragments had penetrated this cloud layer, it is likely that some of the fragments were at least large enough to have passed through Jupiter's outer atmosphere without being burned up.
In the astronomer's argument, the two portions in boldface play which of the following roles?
(A) The first presents a circumstance for which the astronomer offers an explanation; the second is part of that explanation.
(B) The first acknowledges a consideration that weighs against the conclusion of the argument; the second is that conclusion.
(C) The first acknowledges a consideration that weighs against the conclusion of the argument; the second provides evidence in support of that conclusion.
(D) The first provides evidence in support of the conclusion of the argument; the second acknowledges a consideration that weighs against that conclusion.
(E) The first is a judgment advanced in support of the conclusion of the argument; the second is that conclusion.
Since the first is used to support the second, it is the best answer.
4.
prep 07-16. (27621-!-item-!-188;#058&003223)
Most of the world's supply of uranium currently comes from mines. It is possible to extract uranium from seawater, but the cost of doing so is greater than the price that uranium fetches on the world market. Therefore, until the cost of extracting uranium from seawater can somehow be reduced, this method of obtaining uranium is unlikely to be commercially viable.
Which of the following would it be most useful to determine in evaluating the argument?
(A) Whether the uranium in deposits on land is rapidly being depleted
If it is true, people have to obtain uranium from seawater, no matter what the cost of doing so is.
(B) Whether most uranium is used near where it is mined
It cannot help to evaluate the argument.
(C) Whether there are any technological advances that show promise of reducing the cost of extracting uranium from seawater
Even though it seems attractive, it is not better than A. Although there are really some technological advances can reduce the cost, however, if we do not degree, we still cannot evaluate the argument.
(D) Whether the total amount of uranium in seawater is significantly greater than the total amount of uranium on land
It cannot help to evaluate the argument.
(E) Whether uranium can be extracted from freshwater at a cost similar to the cost of extracting it from seawater
Similar cost does not help to evaluate the argument.
作者: pennyz 时间: 2013-2-6 23:26
50s
premise:the society will reduce the power of responsible user,if they eventually find that
(in the long run)
conclusion:company want to live must act responsibly
rephrase:it is longer to realize than company live
C
1:03
premise:if the initial can make great market in America
conclusion:the industry will be in significant producing level
since it doesn't work in the premium condition for the low demand
E(not for sure)
37s
premise:the higher vitamin c we contain,the better function of the lungs
conlusion:smoker can improve their lung function by take in more vitamin C
assumption:the smokers' lungs funtion badly,because they lack of vitamin C
E
1:27
solve a problem
problem:the size of the comet
premise:no sulfer in the comet,but in the outer atmosphere
sulpher inner ,large enough not burned
E
50s
background:possible to get U from seawater,but too expensive
Premise:seek ways to get U from seawater
conclusion:can be economical
evaluation
U get from seawater is same as u we used now
C
对完全今天的大多数题型完全无感!说明漏洞很多!
作者: pennyz 时间: 2013-2-7 13:42
[quote]
B: a company will erode its power if it does not act responsibly. The history of human institution can prove that.
P: the society will reduce the power of companies, if it thinks they are not responsible.
C: A company want to retain its power must be responsible.
Pre: A company actually is very responsible, but it still lose its power./ There is another reason why the company lose its power./ The analogy between companies and human institutions is not proper.
A. This choice actually supports the argument because it prove that the analogy between human institutions and companies is reasonable.
B. Contender. If a company is not responsible but still gain power, then the conclusion may be weakened.
C. “Some companies” cannot stand for “all” companies, so this is an irrelevant comparison.
E. I think this choice is right. Because if a company is responsible but still lose its power, then the conclusion may be weakened.
1.没有注意一个是“wish to retain its power”,另外一个是”retain power”
2.没有发现这里是一个mistaken reversal of the conclusion.
conclusion:wish to retain power→ responsibility
那么reverse一下就是:responsibility→ wish to retain power(假设这里忽略wish的问题)
也就是说“负责”就足以带来“retain power”的结果,选项E中说,负责不足以带来保持实力,等于是weaken了reverse之后的argument,然而weaken mistaken reversal of the conclusion是不能起到weaken原来的argument的作用的。所以错
The Rules of Reversibility
Certain Formal Logic relationships have a natural “reversibility.” Reversibility in the context of Formal Logic means that the relationship between the two variables has exactly the same meaning regardless of which “side” of the relationship is the starting point of your analysis. Statements that are non-reversible have a single “direction,” that is, the relationship between the two variables is not the same.
First, let us examine a relationship that is not reversible:
Starting from the A side, we know that every single A is a B. If we start at B, does the relationship reverse? That is, is every single B an A? No—that would be a Mistaken Reversal. From B’s side, we do not know if every B is an A. Instead, we only know that some B’s are A’s (this inherent inference will be discussed in greater detail in the Inherent versus Additive Inferences section). Thus, the arrow between A and B in the diagram above has a direction: the “all” travels only from A to B and it does not additionally travel from B to A. The relationship is therefore not reversible.
Now, let us examine a reversible relationship. “Some” is a classic example of a reversible statement.
Consider the following example:
Starting from A yields, “Some A’s are B’s” (A some B). Starting from B yields “Some B’s are A’s” (B some A). Because of the nature of “some,” these two statements are functionally identical (if some A’s are B’s, by definition some B’s must also be A’s; alternatively, if some A’s are B’s, then somewhere in the world there is an AB pair, and thus somewhere a B is with an A and we can conclude some B’s are A’s).
Reversible statements are easily identifiable because the relationship symbol is symmetrical and does not include an arrow pointing in a single direction. Non-reversible terms have arrow that point in just one direction.
Reversible Relationships Non-reversible Relationships
The beauty of reversible terms is that you can analyze the relationship from either “side” and still arrive at the same conclusion.
作者: 一粒黄豆。 时间: 2013-3-8 18:04
3/8
1'09'
premise: not use power responsibly-->lose power
conclusion:act responsibly-->business retain power
prephrase: is this a good analogy between business and human institutions?
A strengthen
B irrelevant
C irrelevant
D out of scope
E correct directly overturn the conclusion
40'
background: US and EU are competing in EU market, and EU are grow stronger
premise:US are going to lose part of EU market;US extend US market
conclusion:US can maintain production level
prephrase: assumption loss=extend
C
38'
premise:higher vitamins--> better lung; smokers' lung is worse than that of nonsmokers
conclusion:increase intake of vitamins-->better lung
prephrase:assumption:people will not take more vitamins just because they have better lungs.
E
1'02'
background:comet's fragments may have entered jupiter's atmosphere
premise:evidence+belive-->fragments have sulfur, fragments have no sulfur, layer below jupiter's outer atmosphere does contain sulfur
conclusion:some fragments entered
prephrase:the first part is an evidence and belief, the second part is conclusion, the first part support the second part.
E
27'
background:most u comes from mines
premise: we could extract u from seawater, but the cost is greater than the price of u in the world market
conclusion:untill the cost of extracting u be reduced, we will not use the method
prephrase:gap:will the price of u increase in the future?
A
作者: 一粒黄豆。 时间: 2013-3-8 18:19
发现好多人第一题都错了也~~~
看了解释以后,觉得这个可以用一个经典的理论来解释!
大人通常告诉我们“你一定要好好学习,以后才会有出息”
如果我们说“可是北大毕业的也有找不到工作的啊”这样是无法削弱的
我们要说“可是同桌韩梅梅的爸爸以前是种地的啊,现在是大老板啊!”这个就可以削弱了!
因为 在大人的conclusion里“好好学习” 是“有出息”的必要条件,所以“不好好学习”会导致“没出息”,但是“好好学习”不一定导致“有出息”,所以E通过“结果不成立”--〉“条件不满足”是无法削弱的
这个真心有点难, E长的太像正确答案了,而且是强力直接推翻conclusion的那种,一般不会去细想到底行不行
作者: okplokpl0714 时间: 2013-3-29 08:31
精练:
Speaker:Cb should know avoiding social responsibility-->gradual erosion of power.
DB:not use power-->lose responsible.The law's application to human institutions<--->history
when social not being used responsibly--society ultimately acts to reduce power
CON:Must act responsibly-->retain power
Weaken猜测:有不需要responsibly就可以retain power的方法
A,D无关,BC两个选项看不懂啊。。。。。纠结。。
Answer:E
果然错了。。心里不是很确定。。。
答案E的点评中是说“攻击结论的逆否命题不能伤害到结论”么???求指教!!!
另外求答案B和C的翻译啊。。。语法还是不够好啊。。。TAT
逻辑链:
1.US生产商将大多数太阳能进口到E,但E的生产商人最终还是占有E的大部分市场。
US政府fund initiatives去鼓励太阳能在美国的使用。
如果这些initiatives成功提高了US太阳能需求,US生产商将很可能保持很高的生产水平,
Since-----
猜测:US的太阳能使用率上升空间很大。
排除法。。。无奈选C。。。
但是和我的猜测相距甚远啊。。求解释。。。。。
2.vm in the bloodstream higher----better lung.
smokers' lung<nonsmokers
CON:intake vm--->improve lung smokers' lung
猜测:搭桥:smokers能够吸收vm
Answer:E
3.BF....BF的分析怎么写。。
Answer:E
4.
BG:Most of the world's supply of uranium currently comes from mines
PRE: It is possible to extract uranium from seawater, but the cost of doing so is greater than the price that uranium fetches on the world market
CON: Therefore, until the cost of extracting uranium from seawater can somehow be reduced, this method of obtaining uranium is unlikely to be commercially viable.
Evaluating猜测:extract uranium from seawater的性价比怎么样
Answer:C
好吧。。我的思维又出问题了。。。选A
这题还是有收获的,就是结论没有仔细读,老问题。。。
结论说:除非cost下降,否则from seawater无商业价值
选项A则排除了cost的因素,如果地面的uranium快没了,那么海里的肯定是有价值的。
作者: yiko7 时间: 2013-6-16 21:25
谢谢楼主!
作者: Mint静默 时间: 2013-7-3 00:37
1.
1) 计时:54s
2) Premise:
In the long run, those who do not use power in a manner which society considers responsible will tend to lose it. The “long run” may require decades or even centuries in some instances.
Conclusion:
A business that wishes to retain its power as long as it can must act responsibly.
3) 推测(Weaken):
一个公司可能存活不了decades就倒闭了,所以不存在reduce power的问题
4) 选项分析:
(A) Government institutions are as subject to the
Iron Law of Responsibility as business
institutions.
Irrelevant
(B) Public relations programs can cause society to
consider an institution socially responsible
even when it is not.
Correct
(C) The power of some institutions erodes more
slowly than the power of others, whether they
are socially responsible or not.
无关比较
(D) Since no institution is eternal, every business
will eventually fail.
感觉和我推测的略像,但是是错的
(E) Some businesses that have used power in
socially responsible ways have lost it.
反premise,错
作者: kid381 时间: 2013-7-7 11:37
7/7/13
2-4
116 45’’
Be responsible to the society is the duty of a firm. Do not fulfilling the respect of society will leading firm to lose power in the long time. Though we do not know how long exactly it is, the day of losing power will come for those firm that do less CSR behaviors.
Weaken: generally, a firm has bankrupts for other reasons before the long time condemn comes (B, my thoughts fit D, but D do not weaken the argument, B shows that firm can achieve its object without doing CSR behaviors).
117 45’’
U.S. solar-power industries currently exports most of products to Europe, but the European local manufacturers are growing up and one day they will reclaim the euro market. So U.S. government encourages internal market because: the U.S. firms can at least have locality advantage in their home country (goes for C, coz it really fits my prediction).
118 35’’
High level of vitamins and minerals, then better lung function. Better-off lung function means the amount of air the person can expel in one second. Smokers generally have worse lung function, so the best way to improve it is eat more healthy food.
Assumption: it is the healthy food really that leads to better lung function not the reverse process (strengthen the causal relationship) (E. weaken the causality).
119 boldface 120’’
Goes for A.
120 28’’
Uranium currently comes from mines, to extract uranium from seawater is possible but it is too expensive to cover the costs, so the only chance to utilize this method is to wait for the decreased costs.
Evaluate: Does the Uranium extracted from seawater have a different chemical composition, which is valuable for doing something, as that comes from mines (stress the value of seawater uranium)? A. people have to endure high cost because they deplete deposits of uranium on land.
作者: airsty1020 时间: 2013-8-10 10:27
1.
not use power considered responsible--> lost the power
retain power<--must act responsibly
w:即使act responsibly ,也lost the power. other factors affect people decision
B: responsible caused by PR program, affect people tell responsible or not
so rensponsible x direct affect power
2.
us solar-europe market
europe solar emerge & capture europe market
US gov encourage solar--if succeeed, us maintain production level
since: US & EURO 链接 us market demand = original europe demand
C:US M IN US enjoy same competitive ad
3.
high v & m -->better lung
smoker lung worse than nonsmoker
improve lung <-- intake food rich in v&m
Assum:food works
E:smoking not prevent v&m entering bloodstream
(conclusion是food有效提升lung, 只说smoker/nonsmoker的lung对比,与diet food无关)
4.
1) study finding, and astronomer contrast opinion
2) reason
c
5.
u from m; u possible from s with higher cost
--> from s cost reduce, this method to be viable
e:is it enough u from m; if no m, s have to from s
A
作者: 可可真可爱 时间: 2013-8-11 21:25
Thxxxxxxxxxxxx
作者: Elisha728 时间: 2013-8-27 07:26
balapupu 发表于 2011-11-22 21:28
answer keys:4.This problem is similar in form to the carpet market problem. The conclusionappears at ...
8'55
BCEEA
all correct
作者: m1nt 时间: 2013-9-1 18:37
1. 55’ 汗,好难懂的
Pre: Though the “long run” may require decades or even centuries in some instances, society ultimately acts to reduce power when society thinks it is not being used responsibly
Con: a business that wishes to retain its power as long as it can must act responsibly.
Weaken:
2. 20’
Pre: US exported solar power generators to Euro, Euro capture European market now, US government encourage solar use in US
Con: If these initiatives succeed in increasing the demand for solar-power generators in the United States, United States manufacturers will probably maintain significant production levels
Support
A E Weaken,
C right
B D 无关
3. 20’
Pre: higher level of vitamin, better lung
Con: smokers have worse lung, can improve by intake foods that are rich in vitamin
Assumption: Smoke wont hurt the intake of vitamin in the food
4.
Pre: The fragments themselves almost certainly contained no sulfur, but many astronomers believe that the cloud layer below Jupiter's outer atmosphere does contain sulfur
Con: it is likely that some of the fragments were at least large enough to have passed through Jupiter's outer atmosphere without being burned up.
5. 20’
Pre: Extract U from seawater, cost is higher
Con: until the cost can be reduced, this method is not able to commercially viable
Weaken: other reason to consider to use it
作者: enkyklios 时间: 2013-9-2 08:14
------------------
作者: lyrsilvia 时间: 2013-9-13 20:09
B CEEA
1. 40'
P: In the long run, those who do not use power in a manner which society considers responsible will tend to lose it. society ultimately acts to reduce power when society thinks it is not being used responsibly
C: a business that wishes to retain its power as long as it can must act responsibly.
Weaken: business need power for a short time?
答案:B
(A) Government institutions are as subject to the Iron Law of Responsibility as business institutions.-----government institutions are not discussed
(B) Public relations programs can cause society to consider an institution socially responsible even when it is not.-------means business need not to act responsibly, but just act like responsibLy. correct
(C) The power of some institutions erodes more slowly than the power of others, whether they are socially responsible or not.---------------有erode就必须retain,不管是more slowly
(D) Since no institution is eternal, every business will eventually fail.-----------whether business will fall is irrelevant to the retain of their power
(E) Some businesses that have used power in socially responsible ways have lost it.----------some businesses are not the case.
2. 40'
P: European manufactures capture the european market. US is encouraging use of solar power within the US. If these initiatives succeed in increasing the demand for solar-power generators in US
C: US manufacturers will maintain significant production levels
Since: US manufacturers can enjoy the US market as well
答案:C
(A) some United States manufacturers have been substantially increasing their output over the last several years------------what the past was is irrelevant
(B) the efficiency of solar-power generators in converting energy from the Sun into electric power is not improving as fast as it once did----------efficiency about the product is not discussed
(C) just as European manufacturers enjoy certain competitive advantages in Europe, so do United States manufacturers in the United States-------------correct
(D) European governments are currently undertaking initiatives to stimulate the use of solar power within Europe--------------European governments do=US must do? what European governments do does not affect the US market
(E) the current market for solar-power generators in the United States is very limited----------market is limited≠demand limited, and even the initiatives success, it does not guarantee US manufactures maintain production levels.
3. 27'
P: the higher level of certain vitamins and minerals, the better lung function.
C: increase intake of foods that are rich in helpful vitamins and minerals will improve lung function
Assumption: the intake is effective, the helpful vitamins will be use effectively
答案:E
(A) Smokers are less likely than nonsmokers to have diets that are rich in vitamins and minerals.--------smokers do what is not related to the plan
(B) The lung function of smokers whose diets are rich in those vitamins and minerals is generally better than that of nonsmokers with comparable diets.------------smoker VS nonsmokers, not the case, we are comparing the smokers situation
(C) People whose diets are deficient in those vitamins and minerals do not typically have other health problems in addition to diminished lung function.-----------other health problems are not discussed
(D) Stopping smoking will not typically improve lung function more than any diet changes can.-----------stop smoke VS diet, not the case or the comparison
(E) Smoking does not introduce into the body chemicals that prevent the helpful vitamins and minerals from entering the bloodstream.-------------correct
4. 38'
P: fragments contained no sulfur, layer below J's outer atmosphere does contain sulfur. unprecedented traces of sulfur after the fragments' entry
C: the fragments were at least large enough to have passed through J's outer atmosphere
BOLDFACE: evidence & conclusion
答案:E
(A) The first presents a circumstance for which the astronomer offers an explanation; the second is part of that explanation.-----the first is not a circumstance, circumstance in this case should be the unprecedented traces of sulfur after the fragments' entry.
(B) The first acknowledges a consideration that weighs against the conclusion of the argument; the second is that conclusion.------------"weighs against" is not correct
(C) The first acknowledges a consideration that weighs against the conclusion of the argument; the second provides evidence in support of that conclusion.-------------"weigh against"
(D) The first provides evidence in support of the conclusion of the argument; the second acknowledges a consideration that weighs against that conclusion.-----------the second is not "weigh against"
(E) The first is a judgment advanced in support of the conclusion of the argument; the second is that conclusion.-------------correct
5. 22'
P: cost of extract uranium from seawater is too high
C: until the cost of extracting uranium from seawater be reduced, the method is unlikely to be commercially viable.
Evaluate: cost is the only factors? uranium from mines will deplete in the future?
答案:A
(A) Whether the uranium in deposits on land is rapidly being depleted--------correct
(B) Whether most uranium is used near where it is mined--------where the uranium is used is irrelevant
(C) Whether there are any technological advances that show promise of reducing the cost of extracting uranium from seawater-------------still means that the cost should be reduced
(D) Whether the total amount of uranium in seawater is significantly greater than the total amount of uranium on land------------total amount is not related to the cost
(E) Whether uranium can be extracted from freshwater at a cost similar to the cost of extracting it from seawater---------the comparison between freshwater and the seawater is meaningless.
作者: suiqiji206 时间: 2013-9-17 13:33
P: don’t use power in a manner which society considers responsible ---lose power
C: business , retain power---act responsibily
B is correct
P: US produce S,most of which is exported to EU ; EU begins to produce S ; US increasing the demand
C: US will maintain production
Cis correct
P: vitamins and minerals in the bloodstream----- a person's lung function; The lung function of smokers is significantly worse
C: one way for smokers to improve their lung function is to increase their intake of foods that are rich in these helpful vitamins and minerals.
E
E
P: extracting uranium from seawater costs greater
C: until the cost of extracting uranium from seawater can somehow be reduced, this method of obtaining uranium is unlikely to be commercially viable.
A
作者: xyinghe 时间: 2013-9-17 20:40
1. 44s
B: no social responsibility → no power “long run”
C: business must act responsibly to retain its LT power.
Weaken: 没看懂
Key: C (B)
2. 30a
B: US produce SPG and exported to Europe.
P1: European manu will capture European market
P2: US encourage use of SP in US
C: US will maintain significant production level
Since: 有能力卖给US
Key: C
3. 25s
B: V and M helps lung function
P: lung function: smokers<nonsmokers
C: increase V and M for smokers → improve lung function
Assumption: 吸烟不会阻止吸收V and M
Key: E
4. 38s
B: the comet broke into fragments before entering J’s atmosphere → trace sulfur to exam the size of the fragments
P1: contain no sulfur; but cloud layer contain sulfur
P2: sulfur will seep into outer layer → some fragments passed through J’s outer atmosphere
Boldface: 1. evidence 2. Conclusion
Key: E
5. 24s
B: U comes from mines
P1: can extract U form seawater
P2: cost of extraction > world market
C: not using the method until the cost reduced
Evaluation: 陆地上的U还够不够
Key: A
作者: manqianchen 时间: 2013-9-19 08:15
【精练】
4. Speaker: Contemporary business firms need to
recognize that avoiding social responsibility
leads to the gradual erosion of power. This is
Davis and Blomstrom’s Iron Law of
Responsibility: “In the long run, those who do
not use power in a manner which society
considers responsible will tend to lose it.” The
law’s application to human institutions
certainly stands confirmed by history. Though
the “long run” may require decades or even
centuries in some instances, society ultimately
acts to reduce power when society thinks it is
not being used responsibly. Therefore, a
business that wishes to retain its power as long
as it can must act responsibly.
Which one of the following statements, if true, most
weakens the speaker’s argument?
P1: D and B's Iron Law of responsibility: in the long run, whose who do not use power in a manner which society considers reponsible will tend to lose it
counter premise: Though the long run may require decades or even centuries in some instances, society ultimately acts to reduce power when society thinks it is not being used responsibly
C: a business that wishes to retain its power as long as it can must act responsible
correct: E
(A) Government institutions are as subject to the
Iron Law of Responsibility as business
institutions.
irrelavant: we are talking about business here, not government
(B) Public relations programs can cause society to
consider an institution socially responsible
even when it is not.
(C) The power of some institutions erodes more
slowly than the power of others, whether they
are socially responsible or not.
(D) Since no institution is eternal, every business
will eventually fail.
(E) Some businesses that have used power in
socially responsible ways have lost it.
causal relationship: responsible leads to power
here, it shows that when responsibility exists, power doesnt exist
---bible-weaken
【逻辑链}】
1.prep 07-13.
Which of the following most logically completes the argument?
United States manufacturers currently produce most of the world's solar-power generators--most of which are exported to Europe. However, European manufacturers are emerging and probably will ultimately capture much of the European market. The United States government is funding initiatives intended to encourage use of solar power within the United States. If these initiatives succeed in increasing the demand for solar-power generators in the United States, United States manufacturers will probably maintain significant production levels, since __________.
F1: US currently exported most of the solar energy produced in the states to Europe
F2: E will ultimately capture much of the European market
find the reason why US manufactures will maintatin sig production levels when the funding initiatives succeed
correct: D
(A) some United States manufacturers have been substantially increasing their output over the last several years
(B) the efficiency of solar-power generators in converting energy from the Sun into electric power is not improving as fast as it once did
irrelavant: need to find sth thats related to the government program
(C) just as European manufacturers enjoy certain competitive advantages in Europe, so do United States manufacturers in the United States
(D) European governments are currently undertaking initiatives to stimulate the use of solar power within Europe
(E) the current market for solar-power generators in the United States is very limited
2.prep 07-14.
The higher the level of certain vitamins and minerals in the bloodstream, the better a person's lung function, as measured by the amount of air the person can expel in one second. The lung function of smokers is significantly worse, on average, than that of nonsmokers. Clearly, therefore, one way for smokers to improve their lung function is for them to increase their intake of foods that are rich in these helpful vitamins and minerals.
P1: high vitamins and minerals can better a person's lung function, as measured by the amount of air the person can expel in one second
P2: lung funcion of a smokers is worse on average than that of non smokers
C: one way for smokers to improve lung function is for them to increase their intake of vitamin minerals rich food
correct: E
negate E, will waken the argument
Which of the following is an assumption on which this argument depends?
(A) Smokers are less likely than nonsmokers to have diets that are rich in vitamins and minerals.
(B) The lung function of smokers whose diets are rich in those vitamins and minerals is generally better than that of nonsmokers with comparable diets.
(C) People whose diets are deficient in those vitamins and minerals do not typically have other health problems in addition to diminished lung function.
(D) Stopping smoking will not typically improve lung function more than any diet changes can.
(E) Smoking does not introduce into the body chemicals that prevent the helpful vitamins and minerals from entering the bloodstream.
3.prep 07-15.
Astronomer: Observations of the Shoemaker-Levi comet on its collision course with Jupiter showed that the comet broke into fragments before entering Jupiter's atmosphere in 1994, but they did not show how big those fragments were. In hopes of gaining some indication of the fragments' size, astronomers studied spectrographic analyses of Jupiter's outer atmosphere. These analyses revealed unprecedented traces of sulfur after the fragments' entry.The fragments themselves almost certainly contained no sulfur, but many astronomers believe that the cloud layer below Jupiter's outer atmosphere does contain sulfur. Since sulfur would have seeped into the outer atmosphere if comet fragments had penetrated this cloud layer, it is likely that some of the fragments were at least large enough to have passed through Jupiter's outer atmosphere without being burned up.
correct: E
In the astronomer's argument, the two portions in boldface play which of the following roles?
(A) The first presents a circumstance for which the astronomer offers an explanation; the second is part of that explanation.
(B) The first acknowledges a consideration that weighs against the conclusion of the argument; the second is that conclusion.
(C) The first acknowledges a consideration that weighs against the conclusion of the argument; the second provides evidence in support of that conclusion.
(D) The first provides evidence in support of the conclusion of the argument; the second acknowledges a consideration that weighs against that conclusion.
(E) The first is a judgment advanced in support of the conclusion of the argument; the second is that conclusion.
4.prep 07-16. (27621-!-item-!-188;#058&003223)
Most of the world's supply of uranium currently comes from mines. It is possible to extract uranium from seawater, but the cost of doing so is greater than the price that uranium fetches on the world market. Therefore, until the cost of extracting uranium from seawater can somehow be reduced, this method of obtaining uranium is unlikely to be commercially viable.
P: most of U comes from mines. cost of extracting U from seawater is too high
C: the method of obtaining U from seawater is not commercially viable until the cost of extracting U from seawater can be reduced
correct: A
Which of the following would it be most useful to determine in evaluating the argument?
(A) Whether the uranium in deposits on land is rapidly being depleted
(B) Whether most uranium is used near where it is mined
need to touch on the cost: whether the cost will be a lot higher if the mine is farther away
(C) Whether there are any technological advances that show promise of reducing the cost of extracting uranium from seawater
(D) Whether the total amount of uranium in seawater is significantly greater than the total amount of uranium on land
(E) Whether uranium can be extracted from freshwater at a cost similar to the cost of extracting it from seawater
irrelavant :we are talking about sea water here
作者: 览物之情 时间: 2013-10-6 05:01
1 33s
Premise: in the long run, those who do not use power in a manner which society considers responsible will tend to lose it.”
Conclusion: a business that wishes to retain its power as long as it can must act responsibly.
weakens the speaker’s argument?
因果推理CQ1因果无联系或 一因多果CQ2干扰因素。
P it's not the only factor influence the power?
A) Government institutions are as subject to the Iron Law of Responsibility as business
institutions. 无关
(B) Public relations programs can cause society to consider an institution sociallresponsible
even when it is not. 无关
(C) The power of some institutions erodes more slowly than the power of others, whether they are socially responsible or not. Correct 说明因果无联系
(D) Since no institution is eternal, every business will eventually fail. 无关这里说as long as pissible
(E) Some businesses that have used power in socially responsible ways have lost it. 无关
2 35S
Which of the following most logically completes the argument?
premise1: United States manufacturers currently produce most of the world's solar-power generators--most of which are exported to Europe. However, European manufacturers are emerging and probably will ultimately capture much of the European market.
Premise2 The United States government is funding initiatives intended to encourage use of solar power within the United States.
Conclusion: If these initiatives succeed in increasing the demand for solar-power generators in the United States, United States manufacturers will probably maintain significant production levels, since __________.
因果推理 还是类比推理
CQ1因果联系。如果是类比推理答案就是二者的共同点
P the potential of domestic consumption is large.
(A) some United States manufacturers have been substantially increasing their output over the last several years 和结论无关
(B) the efficiency of solar-power generators in converting energy from the Sun into electric power is not improving as fast as it once did 和结论无关
(C) just as European manufacturers enjoy certain competitive advantages in Europe, so do United States manufacturers in the United States correct
(D) European governments are currently undertaking initiatives to stimulate the use of solar power within Europe 无关
(E) the current market for solar-power generators in the United States is very limited weaken.
对CE选项有些不确定
3 27s
Premise: The higher the level of certain vitamins and minerals in the bloodstream, the better a person's lung function
Conclusion: therefore, one way for smokers to improve their lung function is for them to increase their intake of foods that are rich in these helpful vitamins and minerals.
Which of the following is an assumption on which this argument depends?
因果Cq1 因果联系 CQ2 干扰因素
答案取非要weaken
(A) Smokers are less likely than nonsmokers to have diets that are rich in vitamins and minerals. 不能说明吃vitamins有无效果这个问题
(B) The lung function of smokers whose diets are rich in those vitamins and minerals is generally better than that of nonsmokers with comparable diets. 比较对象不对。If it's compare to smokers without vitamins it might strengthen the conclusion.
(C) People whose diets are deficient in those vitamins and minerals do not typically have other health problems in addition to diminished lung function. 无关,取非不影响结论
(D) Stopping smoking will not typically improve lung function more than any diet changes can. 无关,取非不影响结论
(E) Smoking does not introduce into the body chemicals that prevent the helpful vitamins and minerals from entering the bloodstream. Correct. If smoking prevent the absorption of vitamins then it's useless to do so.
4 60s not very clear
Astronomer: In hopes of gaining some indication of the fragments' size, astronomers studied spectrographic analyses of Jupiter's outer atmosphere. These analyses revealed unprecedented traces of sulfur after the fragments' entry. Premise:The fragments themselves almost certainly contained no sulfur, but many astronomers believe that the cloud layer below Jupiter's outer atmosphere does contain sulfur. Since sulfur would have seeped into the outer atmosphere if comet fragments had penetrated this cloud layer, conclusion:it is likely that some of the fragments were at least large enough to have passed through Jupiter's outer atmosphere without being burned up.
(A) The first presents a circumstance for which the astronomer offers an explanation; the second is part of that explanation.
(B) The first acknowledges a consideration that weighs against the conclusion of the argument; the second is that conclusion.
(C) The first acknowledges a consideration that weighs against the conclusion of the argument; the second provides evidence in support of that conclusion.
(D) The first provides evidence in support of the conclusion of the argument; the second acknowledges a consideration that weighs against that conclusion.
(E) The first is a judgment advanced in support of the conclusion of the argument; the second is that conclusion. Correct
5 28s
Premise: Most of the world's supply of uranium currently comes from mines. It is possible to extract uranium from seawater, but the cost of doing so is greater than the price that uranium fetches on the world market.
Conclusion: Therefore, until the cost of extracting uranium from seawater can somehow be reduced, this method of obtaining uranium is unlikely to be commercially viable.
因果推理:CQ1因果联系CQ2干扰因素
Which of the following would it be most useful to determine in evaluating the argument?产生相反的影响。
(A) Whether the uranium in deposits on land is rapidly being depleted correct. 如果耗尽了必须从seawater提取。
(B) Whether most uranium is used near where it is mined 无关
(C) Whether there are any technological advances that show promise of reducing the cost of extracting uranium from seawater
(D) Whether the total amount of uranium in seawater is significantly greater than the total amount of uranium on land 无关
(E) Whether uranium can be extracted from freshwater at a cost similar to the cost of extracting it from seawater 无关
作者: w.melhere 时间: 2013-10-17 09:54
今天的题目好像有点复杂啊....
作者: w.melhere 时间: 2013-10-17 10:12
1. 48'
2.34'
3.35' 这道题有点没反应过来是假设体啊.找错了key,应该是最后一句话对肺功能不好的改善方法的假设,而非对前面因果的假设. 错选的A应该是有点削弱的,如果nonsmokers不吃这些食物的话,题目中给出的解决办法则不能实行.
4.57' 黑脸还是把握不好,又是天体内容,着实有点晕啦! 看来需要一个黑脸合集好好加强一下!
5.27'
有没有NN可以给小女子一个黑脸讲解或者合集呢? 在此跪谢!!
作者: 笨笨熊vanilla 时间: 2013-10-17 19:30
14. A解释: smoker 吃的少(vitamin和minerals的食物)所以lung不好————重复了题目的P:he higher the level of certain vitamins and minerals in the bloodstream, the better a person's lung function
不知道我解释的对不对~~~~
最近在恶补逻辑,每天做题不定量 就不来签到打卡凑热闹了
但是还是请求楼主和各位逻辑大牛帮我评判哈~ 不要让我出现思想错误。。。。
谢谢大家!!!!
作者: 花生张 时间: 2013-10-19 12:53
计时:51 S
逻辑链:society will reduce irresponsible firms---a business that wishes to retain its power as long
as it can must act responsibly
预测:weaken?
是因为社会的力量,企业才必须负责?还是其他的原因?
选项分析:选B
A 无关
B 第三因素,保留
C 不管负不负责,power都会逐渐消失,但是 as long as possible,能延长就尽量延长
D as long as possible不等于forever,无关
E 无关
计时35S
逻辑链:US 向欧洲出口太阳能发电机---欧洲本地厂商开始挤压市场—提高US本国的需求量—US的企业的产量不会发生变化
预测:complete argument
在欧洲丢掉的市场份额在本国能补回来
选项分析:选E
A 无关
B 无关
C 涉及到结论,保留
D 无关
E 现在的市场开发程度小,所以开发的空间很大 correct
计时:30S
逻辑链:维生素和矿物质对肺有好处---吸烟的人的肺功能较差---多吃含维生素和矿物质的食物
预测:assumption
维生素和矿物质能被人体吸收
吸烟的人的肺跟正常人的肺的差别在于呼吸能力
选项分析:选E
A 削弱
B 无关
C 保留
D 无关
E 保留
C E比较 E对结论有更直接的影响
计时 65S
逻辑链:SL在进入J的大气层之前已经变成了碎片---宇航员要检验这些碎片的大小---研究J的外层大气发现了在碎片进入后有sulfur的痕迹----这些碎片本身没有sulfur,但是J外围大气层下面的云里含有sulfur---因为sulfur可以跟碎片一起进去大气层—所以碎片要足够大到在进去外围大气层之前不burned up
选项分析:选E
计时:26S
逻辑链:U从海水中提取的成本高于U目前的市场价格---所以在成本下降之前,从海水中提取U的方法不太可能被商业
预测:evaluate
U的需求上升,价格也会跟着上升
选项分析:选A
A 保留
B 无关
C 与前提重复
D 无关
E 无关
作者: 紫星当空 时间: 2013-11-4 21:20
4 51秒
P 当代的公司不履行责任会权利丢失
C 所以当代的公司要想retain power就要负责
削弱题 预测:它因导致,即使履行责任也会lost power
A 无关吧
B 不在讨论范围内,无关
C 无关比较
D 正确 所有都会失败----有了责任还是会lost power
E 个别不影响整体
1 59秒
填空题非argument
预测:国内市场比较大
A文中为将来时,于过去无关
B 反相
C 无关比较
D 与欧洲无关
E 正确
2 43秒
P vitamin and minerals 的含量为高,肺功能越好
C 吸烟者提高肺功能要增加vitamin and minerals的食物的摄入
预测:吸烟的人不是吃了不吸收
A 无关比较
B无关比较
C无关
D 没有E 好
E 取非不吸收,削弱所以正确
3 44秒
预测:第一个支持文章论点,第二句main conclusion
选E
应为:群文通篇无大的转折
4 34秒
P从海中抽取uranium很贵
C 除非抽取费用下降,否则经济不可行
预测whether从mines中抽取数量不能满足需求,市场组要大量uranium
A 起到加强和削弱的双重作用
B 无关
C 只有加强作用
D 无关
E 文中没说freshwater 的事
作者: yuehuasunday 时间: 2013-11-5 15:28
Section: 1-3
1.
Background: avoiding social responsibility
leads to the gradual erosion of power.
Premise: In the long run, those who do
not use power in a manner which society
considers responsible will tend to lose it.
Conclusion: Therefore
business that wishes to retain its power as long
as it can must act responsibly.
(A) Unrelated
(B) CORRECT
© Support
(D) Off-topic
(E) Support
2.
Background: United States manufacturers currently produce most of the world's solar-power generators--most of which are exported to Europe.
Premise: However, European manufacturers are emerging and probably will ultimately capture much of the European market. The United States government is funding initiatives intended to encourage use of solar power within the United States.
Conclusion: United States manufacturers will probably maintain significant production levels, since __________.
(A) Unrelated
(B) Unrelated
© Correct
(D) Unrelated
(E) Unrelated
3.
Background: The higher the level of certain vitamins and minerals in the bloodstream, the better a person's lung function, as measured by the amount of air the person can expel in one second.
Premise: The lung function of smokers is significantly worse, on average, than that of nonsmokers.
Conclusion: one way for smokers to improve their lung function is for them to increase their intake of foods that are rich in minerals
(A) unrelated
(B) doesn't help the argument
© unrelated
(D) unrelated
(E) CORRECT
4.
Background: These analyses revealed unprecedented traces of sulfur after the fragments' entry.
Premise: The fragments themselves almost certainly contained no sulfur, but many astronomers believe that the cloud layer below Jupiter's outer atmosphere does contain sulfur.
Conclusion: it is likely that some of the fragments were at least large enough to have passed through Jupiter's outer atmosphere without being burned up.
(A) wrong
(B) wrong
© unrelated
(D) wrong
(E) correct
5.
Background: Most of the world's supply of uranium currently comes from mines.
Premise: It is possible to extract uranium from seawater, but the cost of doing so is greater than the price that uranium fetches on the world market.
Conclusion: Therefore, until the cost of extracting uranium from seawater can somehow be reduced, this method of obtaining uranium is unlikely to be commercially viable.
Answer: A
作者: irenetopia 时间: 2013-11-17 20:27
【精练】4. Speaker: Contemporary business firms need torecognize that avoiding social responsibilityleads to the gradual erosion of power. This isDavis and Blomstrom’s Iron Law ofResponsibility: “In the long run, those who donot use power in a manner which societyconsiders responsible will tend to lose it.”
not being used responsibly. Therefore, abusiness that wishes to retain its power as longas it can must act responsibly.Which one of the following statements, if true, mostweakens the speaker’s argument?
2:00
这题……其实我没看懂……然后……蒙对了……逻辑链:P:大概就是……社会责任对企业的long run是很重要的,一旦它被认为没有责任感,他的地位就保不住了,就等于要垮了(是酱紫吗?酱紫总结对吗?)
C:如果企业要可持续发展就要有开启社会责任模式
问weaken:就是找可以改变因的一个因素,说明这个社会责任模式并不一定要开启,或者可能有个其他什么的帮他搞定了。
(A) Government institutions are as subject to theIron Law of Responsibility as businessinstitutions.——这个说的是政府机构对于企业机构的关系,无关(B) Public relations programs can cause society toconsider an institution socially responsibleeven when it is not.——这个说的公共关系可能考虑一个机构的社会责任当他没有的时候,留着(C) The power of some institutions erodes moreslowly than the power of others, whether theyare socially responsible or not.——一些企业和另一些企业的对比,不相干啊,不要(D) Since no institution is eternal, every businesswill eventually fail.——靠这个好消极,不要(E) Some businesses that have used power insocially responsible ways have lost it.——一些在社会责任里使用权力的企业已经失去了它?不要---bible-weaken
所以我选的B
【逻辑链}】1.prep 07-13. Which of the following most logically completes the argument?United States manufacturers currently produce most of the world's solar-power generators--most of which are exported to Europe. However, European manufacturers are emerging and probably will ultimately capture much of the European market. The United States government is funding initiatives intended to encourage use of solar power within the United States. If these initiatives succeed in increasing the demand for solar-power generators in the United States, United States manufacturers will probably maintain significant production levels, since __________.
1:30(这题选错了,纠结在选项会出现欧洲有关的,真是脑残,一开始选的D)
逻辑链:
P:美国制造全世界最多的太阳能产品,大多数它生产的这个产品出口欧洲
但是控制欧洲市场的往往是欧洲自己生产的产品——因此美国开始鼓励自己国家人用这个
C:如果这个成功的话,美国生产商要保持生产水平,问原因
思路:问原因,这题我也是蒙的,我有病吧晕,选的C,真心发现自己逻辑不好
some United States manufacturers have been substantially increasing their output over the last several years——无关吧(B) the efficiency of solar-power generators in converting energy from the Sun into electric power is not improving as fast as it once did——无关啊(C) just as European manufacturers enjoy certain competitive advantages in Europe, so do United States manufacturers in the United States——跟欧洲对比,因为跟欧洲一样有本土市场,所以要保持生产力?(D) European governments are currently undertaking initiatives to stimulate the use of solar power within Europe——无关(E) the current market for solar-power generators in the United States is very limited——这个都已经很限制了,大量生产木有用啊
2.prep 07-14. The higher the level of certain vitamins and minerals in the bloodstream, the better a person's lung function, as measured by the amount of air the person can expel in one second. The lung function of smokers is significantly worse, on average, than that of nonsmokers. Clearly, therefore, one way for smokers to improve their lung function is for them to increase their intake of foods that are rich in these helpful vitamins and minerals.Which of the following is an assumption on which this argument depends?
1:30
逻辑链:P维他命和矿物质越多,肺功能越好——抽烟者的肺功能平均下来没有不抽烟的人的好。C因此我们要多让吸烟的人吃含有维他命和矿物质的食物,这是提高肺功能的好办法之一
问假设
思路:吃的多是一回事,能不能吸收是另一回事
(A) Smokers are less likely than nonsmokers to have diets that are rich in vitamins and minerals.——所以更要吃……
(B) The lung function of smokers whose diets are rich in those vitamins and minerals is generally better than that of nonsmokers with comparable diets.——无关,吃了同样维生素矿物质食物的吸烟者肺功能比非吸烟者好(C) People whose diets are deficient in those vitamins and minerals do not typically have other health problems in addition to diminished lung function.——other health problems...无关(D) Stopping smoking will not typically improve lung function more than any diet changes can.——进讨论吃或不吃,没讨论新的方法,无关(E) Smoking does not introduce into the body chemicals that prevent the helpful vitamins and minerals from entering the bloodstream.——bingo!不然吃再多也没用
prep 07-15. Astronomer: Observations of the Shoemaker-Levi comet on its collision course with Jupiter showed that the comet broke into fragments before entering Jupiter's atmosphere in 1994, but they did not show how big those fragments were. In hopes of gaining some indication of the fragments' size, astronomers studied spectrographic analyses of Jupiter's outer atmosphere. These analyses revealed unprecedented traces of sulfur after the fragments' entry. The fragments themselves almost certainly contained no sulfur, but many astronomers believe that the cloud layer below Jupiter's outer atmosphere does contain sulfur. Since sulfur would have seeped into the outer atmosphere if comet fragments had penetrated this cloud layer, it is likely that some of the fragments were at least large enough to have passed through Jupiter's outer atmosphere without being burned up.In the astronomer's argument, the two portions in boldface play which of the following roles?1:06
这个题……
我说不出思路
反正就是选E
我反省……
(A) The first presents a circumstance for which the astronomer offers an explanation; the second is part of that explanation.(B) The first acknowledges a consideration that weighs against the conclusion of the argument; the second is that conclusion.(C) The first acknowledges a consideration that weighs against the conclusion of the argument; the second provides evidence in support of that conclusion.(D) The first provides evidence in support of the conclusion of the argument; the second acknowledges a consideration that weighs against that conclusion.(E) The first is a judgment advanced in support of the conclusion of the argument; the second is that conclusion.4.prep 07-16. (27621-!-item-!-188;#058&003223)Most of the world's supply of uranium currently comes from mines. It is possible to extract uranium from seawater, but the cost of doing so is greater than the price that uranium fetches on the world market. Therefore, until the cost of extracting uranium from seawater can somehow be reduced, this method of obtaining uranium is unlikely to be commercially viable.Which of the following would it be most useful to determine in evaluating the argument?1:10
逻辑链:P:大多数的轴来自没空,也有一些来自海
但是海的成本比煤矿多
C:除非成本下降,否则不会开发海里的轴
问评估
思路:
找一个选项,看看是不是能够弥补开发成本,或者说某些行业一定要用到海里的轴
(A) Whether the uranium in deposits on land is rapidly being depleted——bingo!陆地上要被开采完了!一定要用到海里的了!(B) Whether most uranium is used near where it is mined——无关(C) Whether there are any technological advances that show promise of reducing the cost of extracting uranium from seawater——这个价格不是你想降,想降就能降(D) Whether the total amount of uranium in seawater is significantly greater than the total amount of uranium on land——数量再多,用不到也无关啊(E) Whether uranium can be extracted from freshwater at a cost similar to the cost of extracting it from seawater——没提到淡水,滚粗
欢迎光临 ChaseDream (https://forum.chasedream.com/) |
Powered by Discuz! X3.3 |